Edes yksi kreationismia tukeva seikka?

Totuudentorvettaja

Kolme-neljä viikkoa on ollut kreationisteillä aikaa listata kreationismia tukevia faktoja (joita Toni T lupaili) tai näyttöä kreationismin tueksi. Ainoa vaatimus on se, että pitää olla sellainen asia, jota tiede ei ole vielä kumonnut ja pitää perustua tutkimukselliseen näyttöön.

Lista alle kiitos!

-
-
-

102

256

    Vastaukset

    Anonyymi (Kirjaudu / Rekisteröidy)
    5000
    • Totuudentorvettaja

      Ja siis tosiasioita, jotka _tukevat_kreationismia_ ei siis selittelyä, ei evoluutioteorian (olematonta) kritiikkiä, vaan jotain mikä tukee kreationismia ja jota tiede ei ole jo kumonnut.

      Kaikki tietävät kyllä sen, että evoluutioteoriaa on tutkittu ja koeteltu yli sata vuotta ja että koko maailman tiedeyhteisö on tuloksien perusteella tullut siihen tulokseen että evoluutioteoria on käytännöllisesti katsoen tosiasia, mutta nyt siis etsitää edes JOTAIN näyttöä kreationismin tueksi kun te siitä vuosikausia olette puhuneet.

      • toni t

      • Totuudentorvettaja
        toni t kirjoitti:

        Todiste luomisesta - Metatieto todistaa Luojasta - http://sauluslahetys.com/uutiset.html?69

        Todiste evoluutioteoriaa vastaan - Evoluutioteoria kumoutuisi Darwinin mukaan - http://sauluslahetys.com/uutiset.html?70

        Ei mitään linkkejä ilman selityksiä, vaan kerro havainto ja mitä se koskee. Linkkiä voi käyttää lähteenä. Suosittelen kuitenkin sellaisia lähteitä, jotka eivät aiheuta naurunpyrskähdyksiä.


      • toni t kirjoitti:

        Todiste luomisesta - Metatieto todistaa Luojasta - http://sauluslahetys.com/uutiset.html?69

        Todiste evoluutioteoriaa vastaan - Evoluutioteoria kumoutuisi Darwinin mukaan - http://sauluslahetys.com/uutiset.html?70

        Toisessa yhteydessä jo totesin, että biologia ei tunne metatietoa Sauluslähetyksen esittämässä merkityksessä lainkaan. Carter hengenheimolaisineen edustaa tässä vain uskonnollislähtöistä yksityisajattelua, jolle ei tieteessä kannatusta löydy.

        Uskovaisille auktoriteetit ovat välttämättömiä, mutta tieteessä sellaisia ei tunneta. Darwinin ajoista evoluution tutkimus on edistynyt erittäin paljon, eivätkä hänen puolentoista vuosisadan takaiset ajatuksensa edusta nykytiedettä, vaikka Darwinin kehittämä evoluutioteoria sinänsä on tiedemaailmassa käytännössä yksimielisesti todettu täysin paikkansapitäväksi.


      • Totuudentorvettaja
        toni t kirjoitti:

        Todiste luomisesta - Metatieto todistaa Luojasta - http://sauluslahetys.com/uutiset.html?69

        Todiste evoluutioteoriaa vastaan - Evoluutioteoria kumoutuisi Darwinin mukaan - http://sauluslahetys.com/uutiset.html?70

        " Notice that each level of structure within the autopoietic hierarchy depends upon the level below it, but it cannot be explained in terms of that lower level. For example, the transition from out-sourced to on-board power generation depends upon there being an electric motor to run. An electric vacuum cleaner could sit in the cupboard forever without being able to rid itself of its dependence upon an outside source of power—it must be imposed from the level above, for it cannot come from the level below. Likewise, autopoiesis is useless if there is no vacuum cleaner to repair, maintain and reproduce. A vacuum cleaner without autopoietic capability could sit in the cupboard forever without ever attaining to the autopoietic stage—it must be imposed from the level above, as it cannot come from the level below.

        The autopoietic hierarchy is therefore structured in such a way that any kind of naturalistic transition from one level to a higher level would constitute a Polanyi impossibility. That is, the structure at level i is dependent upon the structure at level i-1 but cannot be explained by the structure at that level. So the structure at level i must have been imposed from level i or above."

        http://creation.com/lifes-irreducible-structure-part-1-autopoiesis

        Eli se mitä on, perustuu siihen mitä aiemmin oli (aivan kuten evoluutioteoriassa - ennen silmää oli rypäs valoherkkiä hermosoluja). Ennen kuin oli nykyisen kaltainen silmä, oli jotain yksinkertaisempaa, josta vaihe vaiheelta kehityttiin nykytilanteeseen. Se perusvirhe tässä tehdään, että mitään lisääntymiskyvytöntä ja perimänsä seuraavalle sukupolvelle antamatonta laitetta (pölynimuri) ei voi verrata elävään olentoon, jonka perimä muuttuu sukupolvi toisensa jälkeen.

        Sitten kirjoitus hyppää outoon ja ilman todisteita olevaan olettamaan, että jos olemassaoleva asia/ominaisuus ei selity aiemmalla (yksinkertaisemmalla) "asteella", niin sitä ei voi olla/muodostua. Tässä ollaan taas niissä vanhoissa ja moneen kertaan kumotuissa kreationistien väitteissä, joiden mukaan jokin elin tai rakenne (vaikka silmä) on sellainen, että sen on täytynyt muodostua kerralla nykyisenlaiseksi tai se ei toimisi ollenkaan/toisi mitään etua kantajalleen. Tämä on kuitenkin kumottu jo aikoja sitten.

        Muuten noissa teksteissä häiritsi ylipäätään se, että niissä ei kerrottu ollenkaan siitä, mihin havaintoihin tai tieteellisiin tutkimuksiin kirjoitukset perustuvat. Puhuttiin vain sen ja sen kirjoittajan "erinomaisesta kirjoituksesta". Toki joku voi pitää jonkin kirjoitusta erinomaisena (tai huonona), mutta todistearvoa siinä ei ole.

        Minä jo tuossa mainitsinkin, että tarvitaan jotain sellaista näyttöä, jota ei ole jo kumottu. Tämä nyt käsitelty on pohjimmiltaan samaa vanhaa "jokin elin on liian monimutkainen syntyäkseen tyhjästä ja sen on täytynyt syntyä kerralla nykyisenlaiseksi" -puppua, joka on käsitelty ja kumottu moneen kertaan.

        Mutta hienoa että yritit. Nyt jotain vähän tiukempaa kamaa:
        -
        -
        -


      • jyrtsilö
        RepeRuutikallo kirjoitti:

        Toisessa yhteydessä jo totesin, että biologia ei tunne metatietoa Sauluslähetyksen esittämässä merkityksessä lainkaan. Carter hengenheimolaisineen edustaa tässä vain uskonnollislähtöistä yksityisajattelua, jolle ei tieteessä kannatusta löydy.

        Uskovaisille auktoriteetit ovat välttämättömiä, mutta tieteessä sellaisia ei tunneta. Darwinin ajoista evoluution tutkimus on edistynyt erittäin paljon, eivätkä hänen puolentoista vuosisadan takaiset ajatuksensa edusta nykytiedettä, vaikka Darwinin kehittämä evoluutioteoria sinänsä on tiedemaailmassa käytännössä yksimielisesti todettu täysin paikkansapitäväksi.

        Ja liput korkealle salkoon nyt kreaveljet. Repe meni päissään tunnustamaan Darwinin evoluutioteorian soopaksi.

        Tokihan me olemme tuon tienneet jo paljon ennen evokkeja joten emme me sitä liputa vaan sitä että pakana tunnustaa syntinsä.
        Ja kun kadonnut lammas löytyy niin sitä on syytä juhlia.


      • jyrtsilö
        Totuudentorvettaja kirjoitti:

        " Notice that each level of structure within the autopoietic hierarchy depends upon the level below it, but it cannot be explained in terms of that lower level. For example, the transition from out-sourced to on-board power generation depends upon there being an electric motor to run. An electric vacuum cleaner could sit in the cupboard forever without being able to rid itself of its dependence upon an outside source of power—it must be imposed from the level above, for it cannot come from the level below. Likewise, autopoiesis is useless if there is no vacuum cleaner to repair, maintain and reproduce. A vacuum cleaner without autopoietic capability could sit in the cupboard forever without ever attaining to the autopoietic stage—it must be imposed from the level above, as it cannot come from the level below.

        The autopoietic hierarchy is therefore structured in such a way that any kind of naturalistic transition from one level to a higher level would constitute a Polanyi impossibility. That is, the structure at level i is dependent upon the structure at level i-1 but cannot be explained by the structure at that level. So the structure at level i must have been imposed from level i or above."

        http://creation.com/lifes-irreducible-structure-part-1-autopoiesis

        Eli se mitä on, perustuu siihen mitä aiemmin oli (aivan kuten evoluutioteoriassa - ennen silmää oli rypäs valoherkkiä hermosoluja). Ennen kuin oli nykyisen kaltainen silmä, oli jotain yksinkertaisempaa, josta vaihe vaiheelta kehityttiin nykytilanteeseen. Se perusvirhe tässä tehdään, että mitään lisääntymiskyvytöntä ja perimänsä seuraavalle sukupolvelle antamatonta laitetta (pölynimuri) ei voi verrata elävään olentoon, jonka perimä muuttuu sukupolvi toisensa jälkeen.

        Sitten kirjoitus hyppää outoon ja ilman todisteita olevaan olettamaan, että jos olemassaoleva asia/ominaisuus ei selity aiemmalla (yksinkertaisemmalla) "asteella", niin sitä ei voi olla/muodostua. Tässä ollaan taas niissä vanhoissa ja moneen kertaan kumotuissa kreationistien väitteissä, joiden mukaan jokin elin tai rakenne (vaikka silmä) on sellainen, että sen on täytynyt muodostua kerralla nykyisenlaiseksi tai se ei toimisi ollenkaan/toisi mitään etua kantajalleen. Tämä on kuitenkin kumottu jo aikoja sitten.

        Muuten noissa teksteissä häiritsi ylipäätään se, että niissä ei kerrottu ollenkaan siitä, mihin havaintoihin tai tieteellisiin tutkimuksiin kirjoitukset perustuvat. Puhuttiin vain sen ja sen kirjoittajan "erinomaisesta kirjoituksesta". Toki joku voi pitää jonkin kirjoitusta erinomaisena (tai huonona), mutta todistearvoa siinä ei ole.

        Minä jo tuossa mainitsinkin, että tarvitaan jotain sellaista näyttöä, jota ei ole jo kumottu. Tämä nyt käsitelty on pohjimmiltaan samaa vanhaa "jokin elin on liian monimutkainen syntyäkseen tyhjästä ja sen on täytynyt syntyä kerralla nykyisenlaiseksi" -puppua, joka on käsitelty ja kumottu moneen kertaan.

        Mutta hienoa että yritit. Nyt jotain vähän tiukempaa kamaa:
        -
        -
        -

        Näytä nyt typerä edes yksi fossiili välimuodosta joka on kehittämässä maitoa tuottamatonta nisää kunnon tissiksi. Tai ole hiljaa.

        Tämä koskee tietenkin myös kuulematonta korvaa joka oli tarkoituksella muovautumassa hauen leukaluusta ihmisen korvaluuksi unohtaen sen että ihmisellä pitäisi olla molemmille leukaluillekin kehitysopillinen edeltäjä.

        Vitsi että evokkien putkinököisyys jaksaa yllättää.


      • jyrtsilö kirjoitti:

        Ja liput korkealle salkoon nyt kreaveljet. Repe meni päissään tunnustamaan Darwinin evoluutioteorian soopaksi.

        Tokihan me olemme tuon tienneet jo paljon ennen evokkeja joten emme me sitä liputa vaan sitä että pakana tunnustaa syntinsä.
        Ja kun kadonnut lammas löytyy niin sitä on syytä juhlia.

        Ei ehkä kannata vielä vetää pilettiä keppiin. Taitosi sekä olla ymmärtämättä että ymmärtää väärin alkaa olla jo niin kiusallisen legendaarinen, että joku uskonsisaristasikin on sen saattanut panna merkille.

        No, ehkä tuo oli sentään kohtuuton toive.


      • RepeRuutikallo
        jyrtsilö kirjoitti:

        Näytä nyt typerä edes yksi fossiili välimuodosta joka on kehittämässä maitoa tuottamatonta nisää kunnon tissiksi. Tai ole hiljaa.

        Tämä koskee tietenkin myös kuulematonta korvaa joka oli tarkoituksella muovautumassa hauen leukaluusta ihmisen korvaluuksi unohtaen sen että ihmisellä pitäisi olla molemmille leukaluillekin kehitysopillinen edeltäjä.

        Vitsi että evokkien putkinököisyys jaksaa yllättää.

        >Näytä nyt typerä edes yksi fossiili välimuodosta joka on kehittämässä maitoa tuottamatonta nisää kunnon tissiksi.

        Jyrtsin peräkammarin paleontologiassa pehmytkudokset näköjään fossiloituvat omituisen vilkkaasti.


      • puolimutkarrrrr
        jyrtsilö kirjoitti:

        Ja liput korkealle salkoon nyt kreaveljet. Repe meni päissään tunnustamaan Darwinin evoluutioteorian soopaksi.

        Tokihan me olemme tuon tienneet jo paljon ennen evokkeja joten emme me sitä liputa vaan sitä että pakana tunnustaa syntinsä.
        Ja kun kadonnut lammas löytyy niin sitä on syytä juhlia.

        Taitaapi jyr olla jo perjantaipullonsa kanssa kovinkin sinut tai sitte jyrillä ei oo vielä vappuputki päättyny. Vai olisko sittenki "Thaikkulasta" mukaan saadut pillerit jääneet ottamatta.

        Täytyy nimittäin olla tavalla tai toisella aivot soosissa, jos Repen kommentin noin tulkitsee.


      • jyrrrrrty
        puolimutkarrrrr kirjoitti:

        Taitaapi jyr olla jo perjantaipullonsa kanssa kovinkin sinut tai sitte jyrillä ei oo vielä vappuputki päättyny. Vai olisko sittenki "Thaikkulasta" mukaan saadut pillerit jääneet ottamatta.

        Täytyy nimittäin olla tavalla tai toisella aivot soosissa, jos Repen kommentin noin tulkitsee.

        Repe sanoo samassa virkkeessä että Darwinin teoria ei ole ja on voimassa, joten kyllä se perjantaiputeli taitaa olla tuon sekopään kaalissa.


      • jyrikop
        RepeRuutikallo kirjoitti:

        >Näytä nyt typerä edes yksi fossiili välimuodosta joka on kehittämässä maitoa tuottamatonta nisää kunnon tissiksi.

        Jyrtsin peräkammarin paleontologiassa pehmytkudokset näköjään fossiloituvat omituisen vilkkaasti.

        Tätäpä juuri tarkoitin, eli ei ole mitään näyttöä siitä että hikirauhasesta kehittyi tissi.

        Kyse on vain siitä evoluutioteorian on väistämätön pakko kehitellä nisille edes jonkinlainen tarina, ja kun uskottavaa on mahdotonta kehittää niin sitten jää vaihtoehdoksi puhua täyttä pas.kaa.


      • jyrikop kirjoitti:

        Tätäpä juuri tarkoitin, eli ei ole mitään näyttöä siitä että hikirauhasesta kehittyi tissi.

        Kyse on vain siitä evoluutioteorian on väistämätön pakko kehitellä nisille edes jonkinlainen tarina, ja kun uskottavaa on mahdotonta kehittää niin sitten jää vaihtoehdoksi puhua täyttä pas.kaa.

        Ahaa. Eli olet sitä mieltä, että jos jokin epäsuorien todisteiden perusteella ilmeinen evolutiivinen juttu ei käytännössä voi fossiloitua kuin äärimmäisen erikoisissa ja harvinaisissa oloissa ja suoraa fossiilinäyttöä sille ei siksi löydy, sitä ei saman tien voi olla tapahtunutkaan.

        Tiede ei toimi ihan noin ainakaan valtakunnansalien ulkopuolella.


      • jyrrrrrty kirjoitti:

        Repe sanoo samassa virkkeessä että Darwinin teoria ei ole ja on voimassa, joten kyllä se perjantaiputeli taitaa olla tuon sekopään kaalissa.

        Jos halustasi ja kyvystäsi keskustella ja ymmärtää mitä sinulle sanotaan ei olisi negatiivista näyttöä vuoren verran ennestään, tämä posketon repliikki olisi ollut viimeinen naula arkkuusi.

        Kyllähän se arkku on ollut valmis jo kauan sitten, ja ihan itse olet sen kyhännyt.


      • ornitologi2
        RepeRuutikallo kirjoitti:

        Toisessa yhteydessä jo totesin, että biologia ei tunne metatietoa Sauluslähetyksen esittämässä merkityksessä lainkaan. Carter hengenheimolaisineen edustaa tässä vain uskonnollislähtöistä yksityisajattelua, jolle ei tieteessä kannatusta löydy.

        Uskovaisille auktoriteetit ovat välttämättömiä, mutta tieteessä sellaisia ei tunneta. Darwinin ajoista evoluution tutkimus on edistynyt erittäin paljon, eivätkä hänen puolentoista vuosisadan takaiset ajatuksensa edusta nykytiedettä, vaikka Darwinin kehittämä evoluutioteoria sinänsä on tiedemaailmassa käytännössä yksimielisesti todettu täysin paikkansapitäväksi.

        Tuo väite ei esim. pidä paikkaansa. Evoluutioteoria on ja pysyy teoriana. Sen sijaan uskontona se voi ihan hyvin puoltaa paikkaansa.


      • ornitologi2 kirjoitti:

        Tuo väite ei esim. pidä paikkaansa. Evoluutioteoria on ja pysyy teoriana. Sen sijaan uskontona se voi ihan hyvin puoltaa paikkaansa.

        Voi neiti hyvä, totta kai evoluutioteoria on ja pysyy tieteellisenä teoriana. Uskonnoksi se on kelvoton, kun se perustuu tutkimukseen ja uusi tieto muuttaa sen yksityiskohtia. Siinä ei myöskään ole mitään palvottavaa, eikä se lupaa ihmiselle yhtikäs mitään kivaa tai uhkaile millään.


      • jyrtsilö kirjoitti:

        Näytä nyt typerä edes yksi fossiili välimuodosta joka on kehittämässä maitoa tuottamatonta nisää kunnon tissiksi. Tai ole hiljaa.

        Tämä koskee tietenkin myös kuulematonta korvaa joka oli tarkoituksella muovautumassa hauen leukaluusta ihmisen korvaluuksi unohtaen sen että ihmisellä pitäisi olla molemmille leukaluillekin kehitysopillinen edeltäjä.

        Vitsi että evokkien putkinököisyys jaksaa yllättää.

        Pitääkö olla fossiili, vai kelpaako ihan elävä yksilö? Vesinokkaeläin erittää maitoa, mutta sillä ei ole tissiä.


      • Anonyymi
        RepeRuutikallo kirjoitti:

        Toisessa yhteydessä jo totesin, että biologia ei tunne metatietoa Sauluslähetyksen esittämässä merkityksessä lainkaan. Carter hengenheimolaisineen edustaa tässä vain uskonnollislähtöistä yksityisajattelua, jolle ei tieteessä kannatusta löydy.

        Uskovaisille auktoriteetit ovat välttämättömiä, mutta tieteessä sellaisia ei tunneta. Darwinin ajoista evoluution tutkimus on edistynyt erittäin paljon, eivätkä hänen puolentoista vuosisadan takaiset ajatuksensa edusta nykytiedettä, vaikka Darwinin kehittämä evoluutioteoria sinänsä on tiedemaailmassa käytännössä yksimielisesti todettu täysin paikkansapitäväksi.

        Niin kaukana faktoista kuin olla voi tuo roskatiede evoluutio.....


    • xyfg

      Tukevia seikkoja ei tule kun niitä ei ole, yksinkertaista.

    • Totuudentorvettaja

      Eikö mitään? Toni T:llä on kokonainen internetsivusto täynnä pa.. kreationismia. Luulisi noin kattavasta kokoelmasta löytyvän edes yksi asia. Jään odottamaan.

      • jatka vain

        Veljeni. Kretujen teoria tuskin on todisteissaan aukoton, mutta uskovat ihmiset eivät perustakaan uskoaan siihen, että kaiken voisi selittää täydellisesti. Uskovat ajattelevat, ettei heidän älykkyytensä ylety heidän luojansa tasolle, ja niinpä elämän selittäminen perustuu pitkälti uskoon korkeammasta voimasta, joka ohjailee kaikkea elämää.

        Ongelmasi on se, että vaikka kuinka liputat/liputatte evoluutioteorian puolesta, ei sekään loogisuudestaan huolimatta ole täysin aukoton. Ja siitä puuttuvat elämän merkityksellisyys ja ihmisen syvempi tietoisuus, jotka vain jumaluus voi selittää. Mutta sinnikäs olet, sen myönnän. Mukava näitä on lueskella joutessaan.


      • jatka vain kirjoitti:

        Veljeni. Kretujen teoria tuskin on todisteissaan aukoton, mutta uskovat ihmiset eivät perustakaan uskoaan siihen, että kaiken voisi selittää täydellisesti. Uskovat ajattelevat, ettei heidän älykkyytensä ylety heidän luojansa tasolle, ja niinpä elämän selittäminen perustuu pitkälti uskoon korkeammasta voimasta, joka ohjailee kaikkea elämää.

        Ongelmasi on se, että vaikka kuinka liputat/liputatte evoluutioteorian puolesta, ei sekään loogisuudestaan huolimatta ole täysin aukoton. Ja siitä puuttuvat elämän merkityksellisyys ja ihmisen syvempi tietoisuus, jotka vain jumaluus voi selittää. Mutta sinnikäs olet, sen myönnän. Mukava näitä on lueskella joutessaan.

        Sisäistähän mukavasti joutessasi tämä: elämän merkityksellisyys on subjektiivinen käsite.

        Ja vastaahan mukavasti joutessasi tähän: Mitä tarkoitat tarkalleen ihmisen syvemmällä tietoisuudella, ja miksi vain jumaluus voisi sen selittää?


    • ”Edes yksi kreationismia tukeva seikka?”

      Yksi sellainen voisi olla, että järjestelmien on pakko olla samaan aikaan olemassa, jotta ne toimisivat.
      Voisi ajatella asiaa käänteisesti: pysyykö ihminen olevaisena, jos tältä poistetaan maksa, munuaiset, keuhkot, aivot, verenkierto, aistit?

      Kukaan tuskin väittää, että olemassaolo olisi itsesyntyistä, sattuman hallintaa elottomasta materiasta. Älykään suunnittelijan avulla sen sijaan voi hyvinkin luoda olevaisuutta. Tosin ihminen ei siihen kykene.

      • Hiski+naapurin.kissa

        Aukotonta todistelua: Sami ei ymmärrä, siis Jumala.


      • Hiski+naapurin.kissa

        Lainauslouhintaa: "pysyykö ihminen olevaisena, jos tältä poistetaan aistit?"

        Helen Keller.


      • Totuudentorvettaja

        "Yksi sellainen voisi olla, että järjestelmien on pakko olla samaan aikaan olemassa, jotta ne toimisivat.
        Voisi ajatella asiaa käänteisesti: pysyykö ihminen olevaisena, jos tältä poistetaan maksa, munuaiset, keuhkot, aivot, verenkierto, aistit?"

        Nykymuodossa pitää olla, mutta aiemmin kun ihmisen evoluutioaskelmia mennään riittävän kauas ajassa taaksepäin, kohti yksinkertaisempaa esi-isää, näin ei ollut. Otetaan vertailun vuoksi esimerkiksi madot. Joillakin madoilla on verenkierto, toisilla ei, useilla madoilla ei ole sydäntä ollenkaan, vaikka on verenkierto. Myöskään keuhkoja ei useinkaan ole. Hermosto sen sijaan löytyy ja aivojenkin alkeellinen versio tikapuuhermoston jatkeena. Yllättikö tieto, että verenkiertoelmistöstä huolimatta osalla madoista ei ole sydäntä tai keuhkoja?

        "Kukaan tuskin väittää, että olemassaolo olisi itsesyntyistä"

        En tiedä mitä kukakin väittää, mutta alkaa näyttää siltä, että se olisi ollut hyvinkin mahdollista (jos elämästä puhut), vaikkei kukaan tietäisi onko se itsestään alkanut vai ei.


      • jyrtelö
        Totuudentorvettaja kirjoitti:

        "Yksi sellainen voisi olla, että järjestelmien on pakko olla samaan aikaan olemassa, jotta ne toimisivat.
        Voisi ajatella asiaa käänteisesti: pysyykö ihminen olevaisena, jos tältä poistetaan maksa, munuaiset, keuhkot, aivot, verenkierto, aistit?"

        Nykymuodossa pitää olla, mutta aiemmin kun ihmisen evoluutioaskelmia mennään riittävän kauas ajassa taaksepäin, kohti yksinkertaisempaa esi-isää, näin ei ollut. Otetaan vertailun vuoksi esimerkiksi madot. Joillakin madoilla on verenkierto, toisilla ei, useilla madoilla ei ole sydäntä ollenkaan, vaikka on verenkierto. Myöskään keuhkoja ei useinkaan ole. Hermosto sen sijaan löytyy ja aivojenkin alkeellinen versio tikapuuhermoston jatkeena. Yllättikö tieto, että verenkiertoelmistöstä huolimatta osalla madoista ei ole sydäntä tai keuhkoja?

        "Kukaan tuskin väittää, että olemassaolo olisi itsesyntyistä"

        En tiedä mitä kukakin väittää, mutta alkaa näyttää siltä, että se olisi ollut hyvinkin mahdollista (jos elämästä puhut), vaikkei kukaan tietäisi onko se itsestään alkanut vai ei.

        Torvettajan suuri viisaus: Absoluuttisesta tyhjyydestä syntyi maailmankaikkeus.


        Katsohan tuota viisautta kirjain kirjaimelta. Siis paperille voi kirjoittaa ihan mitä tahansa palturia, se käy helposti.


      • jyrtelö kirjoitti:

        Torvettajan suuri viisaus: Absoluuttisesta tyhjyydestä syntyi maailmankaikkeus.


        Katsohan tuota viisautta kirjain kirjaimelta. Siis paperille voi kirjoittaa ihan mitä tahansa palturia, se käy helposti.

        Ja jyrtelön viisautta on näemmä olla ulottamatta hehkuttamamansa pointin (kirjaimet paperilla) väkevyyttä Raamattuun.


      • khaosmonaut kirjoitti:

        Ja jyrtelön viisautta on näemmä olla ulottamatta hehkuttamamansa pointin (kirjaimet paperilla) väkevyyttä Raamattuun.

        Oho! Aloin vapun jälkihehkussa puhua hehkuttamisesta kielillä.


      • Hiski+naapurin.kissa kirjoitti:

        Lainauslouhintaa: "pysyykö ihminen olevaisena, jos tältä poistetaan aistit?"

        Helen Keller.

        "Lainauslouhintaa: "pysyykö ihminen olevaisena, jos tältä poistetaan aistit?""

        Ihminen pysyy voimassa, koska Jumala loi ihmisen kuvakseen. Alussa kaikki oli kunnossa.


      • Totuudentorvettaja kirjoitti:

        "Yksi sellainen voisi olla, että järjestelmien on pakko olla samaan aikaan olemassa, jotta ne toimisivat.
        Voisi ajatella asiaa käänteisesti: pysyykö ihminen olevaisena, jos tältä poistetaan maksa, munuaiset, keuhkot, aivot, verenkierto, aistit?"

        Nykymuodossa pitää olla, mutta aiemmin kun ihmisen evoluutioaskelmia mennään riittävän kauas ajassa taaksepäin, kohti yksinkertaisempaa esi-isää, näin ei ollut. Otetaan vertailun vuoksi esimerkiksi madot. Joillakin madoilla on verenkierto, toisilla ei, useilla madoilla ei ole sydäntä ollenkaan, vaikka on verenkierto. Myöskään keuhkoja ei useinkaan ole. Hermosto sen sijaan löytyy ja aivojenkin alkeellinen versio tikapuuhermoston jatkeena. Yllättikö tieto, että verenkiertoelmistöstä huolimatta osalla madoista ei ole sydäntä tai keuhkoja?

        "Kukaan tuskin väittää, että olemassaolo olisi itsesyntyistä"

        En tiedä mitä kukakin väittää, mutta alkaa näyttää siltä, että se olisi ollut hyvinkin mahdollista (jos elämästä puhut), vaikkei kukaan tietäisi onko se itsestään alkanut vai ei.

        ”Nykymuodossa pitää olla, mutta aiemmin kun ihmisen evoluutioaskelmia mennään riittävän kauas ajassa taaksepäin, kohti yksinkertaisempaa esi-isää”

        Elämä ei ole yksinkertaista ollut missään vaiheessa. Oletus yksinkertaisesta elämästä on elämän näkökulmasta outo.
        Jos elämä on noin yksinkertaista, niin miksi ihminen ei kykene luomaan yksinkertaista solua? Vaikka sellaista solua, jota sinä väität yksinkertaiseksi.


      • Totuudentorvettaja
        sami-a kirjoitti:

        ”Nykymuodossa pitää olla, mutta aiemmin kun ihmisen evoluutioaskelmia mennään riittävän kauas ajassa taaksepäin, kohti yksinkertaisempaa esi-isää”

        Elämä ei ole yksinkertaista ollut missään vaiheessa. Oletus yksinkertaisesta elämästä on elämän näkökulmasta outo.
        Jos elämä on noin yksinkertaista, niin miksi ihminen ei kykene luomaan yksinkertaista solua? Vaikka sellaista solua, jota sinä väität yksinkertaiseksi.

        "Elämä ei ole yksinkertaista ollut missään vaiheessa."

        Kirjoitin useampia aloituksia, mutta en keksinyt mitään sopivaa vastausta noin hölmöön kommenttiin. Voin toki vastata kreationistien omilla sanoilla ja kysyä, että olitko sinä paikalla todistamassa, kun elämä sai alkunsa, ettei sen ollut yksinkertaista? Mutta siltikin asia on merkityksetön kirjoitukseni kannalta, koska yksinkertaisempi ei edes tarkoita yksinkertaista, vaan yksinkertaisempaa - siis vähemmän monimutkaista.

        "Jos elämä on noin yksinkertaista, niin miksi ihminen ei kykene luomaan yksinkertaista solua?"

        Vastauksesi ei myöskään liity mitenkään esittämääsi kysymykseen, johon vastasin. Hyppäsit lukemattomia sukupolvia aiempaan asiaan. Onko liikaa pyydetty, että pysyisitte siinä asiassa, jota käsitellään? Väitit että ihmisen elimien piti olla olemassa yhtä aikaa toimiakseen. Osoitin että on olemassa elämää, jossa kaikkia elimiä ei ole yhtäaikaisesti - esimerkiksi verisuonet ilman sydäntä. Sitten hyppäsit soluihin yrittäen siirtää syrjään sen, että olit väärässä.


      • Totuudentorvettaja kirjoitti:

        "Elämä ei ole yksinkertaista ollut missään vaiheessa."

        Kirjoitin useampia aloituksia, mutta en keksinyt mitään sopivaa vastausta noin hölmöön kommenttiin. Voin toki vastata kreationistien omilla sanoilla ja kysyä, että olitko sinä paikalla todistamassa, kun elämä sai alkunsa, ettei sen ollut yksinkertaista? Mutta siltikin asia on merkityksetön kirjoitukseni kannalta, koska yksinkertaisempi ei edes tarkoita yksinkertaista, vaan yksinkertaisempaa - siis vähemmän monimutkaista.

        "Jos elämä on noin yksinkertaista, niin miksi ihminen ei kykene luomaan yksinkertaista solua?"

        Vastauksesi ei myöskään liity mitenkään esittämääsi kysymykseen, johon vastasin. Hyppäsit lukemattomia sukupolvia aiempaan asiaan. Onko liikaa pyydetty, että pysyisitte siinä asiassa, jota käsitellään? Väitit että ihmisen elimien piti olla olemassa yhtä aikaa toimiakseen. Osoitin että on olemassa elämää, jossa kaikkia elimiä ei ole yhtäaikaisesti - esimerkiksi verisuonet ilman sydäntä. Sitten hyppäsit soluihin yrittäen siirtää syrjään sen, että olit väärässä.

        ”Mutta siltikin asia on merkityksetön kirjoitukseni kannalta, koska yksinkertaisempi ei edes tarkoita yksinkertaista, vaan yksinkertaisempaa - siis vähemmän monimutkaista.”

        Pyrit siis pitämään yllä kehityspotentiaalia. En usko, että on olemassa kehityspotentiaalia, vaan ainoastaan olemassa olon potentiaalia. Kaikki potentiaaliset olevaiset ovat jo olemassa. Siitä, mitä ei ole, voit jatkaa sinä…

        ”Onko liikaa pyydetty, että pysyisitte siinä asiassa, jota käsitellään?”

        Ei ole.

        ”Väitit että ihmisen elimien piti olla olemassa yhtä aikaa toimiakseen. Osoitin että on olemassa elämää, jossa kaikkia elimiä ei ole yhtäaikaisesti - esimerkiksi verisuonet ilman sydäntä.”

        Ainakin minun mielestä kaikki ihmisen elimet ja järjestelmät on oltava yhtä aikaa toimivia. Ajatukseni ei pitäisi olla epätieteellinen. Mitään evoluutioteorian kuvaamaa toleranssia tuskin on. Voit toki ajatella, että on, mutta se ei ole mielekästä.


      • sami-a kirjoitti:

        ”Nykymuodossa pitää olla, mutta aiemmin kun ihmisen evoluutioaskelmia mennään riittävän kauas ajassa taaksepäin, kohti yksinkertaisempaa esi-isää”

        Elämä ei ole yksinkertaista ollut missään vaiheessa. Oletus yksinkertaisesta elämästä on elämän näkökulmasta outo.
        Jos elämä on noin yksinkertaista, niin miksi ihminen ei kykene luomaan yksinkertaista solua? Vaikka sellaista solua, jota sinä väität yksinkertaiseksi.

        >Jos elämä on noin yksinkertaista, niin miksi ihminen ei kykene luomaan yksinkertaista solua?

        Mistä ihmeestä sinä tämän voit tietää?

        Katsotaanpa nyt aluksi vaikka sata vuotta tästä eteenpäin. Yritystä kun on ollut vasta kovin lyhyen aikaa ja konstit tehtävän vaativuuteen nähden suhteellisen alkeellisia.

        Ai mutta nyt taisinkin unohtaa, että kreationistisessa ajattelussa ihmistieteen kehityksen pysyvä huippu on saavutettu sinä hetkenä jota keskustelua käytäessä eletään (Ruutikallon laki).


      • Totuudentorvettaja
        sami-a kirjoitti:

        ”Mutta siltikin asia on merkityksetön kirjoitukseni kannalta, koska yksinkertaisempi ei edes tarkoita yksinkertaista, vaan yksinkertaisempaa - siis vähemmän monimutkaista.”

        Pyrit siis pitämään yllä kehityspotentiaalia. En usko, että on olemassa kehityspotentiaalia, vaan ainoastaan olemassa olon potentiaalia. Kaikki potentiaaliset olevaiset ovat jo olemassa. Siitä, mitä ei ole, voit jatkaa sinä…

        ”Onko liikaa pyydetty, että pysyisitte siinä asiassa, jota käsitellään?”

        Ei ole.

        ”Väitit että ihmisen elimien piti olla olemassa yhtä aikaa toimiakseen. Osoitin että on olemassa elämää, jossa kaikkia elimiä ei ole yhtäaikaisesti - esimerkiksi verisuonet ilman sydäntä.”

        Ainakin minun mielestä kaikki ihmisen elimet ja järjestelmät on oltava yhtä aikaa toimivia. Ajatukseni ei pitäisi olla epätieteellinen. Mitään evoluutioteorian kuvaamaa toleranssia tuskin on. Voit toki ajatella, että on, mutta se ei ole mielekästä.

        "Pyrit siis pitämään yllä...blaablaablaa"

        Minä en pyri mihinkään, vaan puhun siitä mitä on havaittu. Sinulla(kaan) ei ole tarvittavaa koulutusta, osaamista, eikä näyttöä jonka avulla voisit kumota tieteen.

        "Ainakin minun mielestä kaikki ihmisen elimet ja järjestelmät on oltava yhtä aikaa toimivia."

        Viisasteletko sinä, vai etkö tosissaan ymmärtänyt mitä tarkoittaa "Nykymuodossa pitää olla, mutta aiemmin kun ihmisen evoluutioaskelmia mennään riittävän kauas ajassa taaksepäin, kohti yksinkertaisempaa esi-isää, näin ei ollut." Tarkennan että näin ei ole täytynyt olla.


      • Mutta miksi kaikki eläimet toimivat samalla tavalla kuin ihmiset.. maksalla sama tarkoitus elimistössä samaten, munuaisilla ja keuhkoilla.. Perusanatomia lajien välillä on hämmästyttävän samankaltainen, esimerkiksi lääkeaineiden käyttömäärä per kilo on hyvin samankaltainen eri eläinlajeilla. Perin kiusallista kreationismin kannalta.


    • Totuudentorvettaja

      "Kretujen teoria tuskin on todisteissaan aukoton"

      En kysele tässä aukottoman teorian perään, vaan ylipäätään JOTAIN osoitusta siitä, että kreationismi olisi muutakin kuin uskon ja mielikuvituksen tuotetta. Siis jotain näyttöä sen tueksi.

      "Ongelmasi on se, että vaikka kuinka liputat/liputatte evoluutioteorian puolesta, ei sekään loogisuudestaan huolimatta ole täysin aukoton."

      Ei, mutta tähän mennessä tilanne evoluutioteorian osalta on kuitenkin se, että olemassaolevat havainnot sopivat sen kanssa yhteen, eikä mitään sellaista havaintoa tai havaintoja ole tehty, jotka eivät kertakaikkiaan sopisi yhteen sen kanssa. Myös sellaisten tieteenalojen havainnot, jotka eivät suoranaisesti liity evoluutioteoriaan/biologiaan, sopivat sen kanssa yhteen - esimerkiksi havainnot maapallon ja maailmankaikkeuden iästä.

      "Ja siitä puuttuvat elämän merkityksellisyys ja ihmisen syvempi tietoisuus, jotka vain jumaluus voi selittää."

      Elämä ei edellytä, että sillä olisi merkitys. Jos muurahainen syntyy ja liiskataan, mikä sen elämän merkitys on? Tietoisuus taas on hyvin kiinteästi yhteydessä aivoihin. Siihen voidaan vaikuttaa lääkkeillä, leikkauksella/tapaturmilla, sähköllä ja niin edelleen. Aivojen toimintaa voidaan myös mitata ja aivojen osien vaikutus persoonallisuuteen tunnetaan jo kohtuullisesti.

      Kreationistit ovat toki vapaita uskomaan mitä haluavat, mutta on loukkaavaa että kreationismi esitetään tieteenä tai tieteen kaltaisena, koska sitä se ei ole. Se ei ole teoria tieteellisessä merkityksessä eikä sen tueksi ei ole olemassa näyttöä. Maailma voi olla luotu, siihen evoluutioteoria ei ota kantaa, mutta on epärehellistä väittää maailman näyttävän tuhansia vuosia vanhalta tai kiistää tieteen fossiili- ja dna-löydöt tai lajiutumisesimerkit. Maailma voi olla luotu, vaikka elämä sen olisikin kehittyvää ja lajiutuvaa. Tämä on kuitenkin se asia, jonka kreationistit jyrkästi kiistävät vedoten Raamatun kirjaimelliseen tulkintaan.

      • jyrtelö

        Sao sinä torvi yksi oikea todiste evoluution puolesta, ja tieteellisten perustelujen kera niin saadaan piste tälle ketjulle.


        On se kumma että tässä kohdassa ketju aina jumittuu ja alkaa evokkien herjanheitto.

        Evokki ajattelee töpöaivoillaan tässä kohdassa näin: No enhän minä mitään osaa vastasta mutta jos haukun kysyjän kuolaavaksi vajakiksi niin huomio kiinnittyy minun tietämättömyydestäni poispäin ja olen kuivilla. Eli porukat luulevat yhä että olen fiksu, vaikka en oikeasti mitään mistään ymmärräkään.

        Mutta näinhän se menee, elämä on evokeille selviytymistaistelua jossa kaikki keinot ovat sallittuja ja varsinkaan moraalia ei tunneta.


      • jyrtelö kirjoitti:

        Sao sinä torvi yksi oikea todiste evoluution puolesta, ja tieteellisten perustelujen kera niin saadaan piste tälle ketjulle.


        On se kumma että tässä kohdassa ketju aina jumittuu ja alkaa evokkien herjanheitto.

        Evokki ajattelee töpöaivoillaan tässä kohdassa näin: No enhän minä mitään osaa vastasta mutta jos haukun kysyjän kuolaavaksi vajakiksi niin huomio kiinnittyy minun tietämättömyydestäni poispäin ja olen kuivilla. Eli porukat luulevat yhä että olen fiksu, vaikka en oikeasti mitään mistään ymmärräkään.

        Mutta näinhän se menee, elämä on evokeille selviytymistaistelua jossa kaikki keinot ovat sallittuja ja varsinkaan moraalia ei tunneta.

        >Mutta näinhän se menee, elämä on evokeille selviytymistaistelua jossa kaikki keinot ovat sallittuja ja varsinkaan moraalia ei tunneta.

        En nyt kutsuisi tätä päivittäistä teikäläisten henkiselle pimeydelle hihittelyä varsinaisesti selviytymistaisteluksi. :D

        Suhteellisuudentajusi taitaa olla yhtä kankaalla kuin koko kretuoppi?


      • Totuudentorvettaja
        jyrtelö kirjoitti:

        Sao sinä torvi yksi oikea todiste evoluution puolesta, ja tieteellisten perustelujen kera niin saadaan piste tälle ketjulle.


        On se kumma että tässä kohdassa ketju aina jumittuu ja alkaa evokkien herjanheitto.

        Evokki ajattelee töpöaivoillaan tässä kohdassa näin: No enhän minä mitään osaa vastasta mutta jos haukun kysyjän kuolaavaksi vajakiksi niin huomio kiinnittyy minun tietämättömyydestäni poispäin ja olen kuivilla. Eli porukat luulevat yhä että olen fiksu, vaikka en oikeasti mitään mistään ymmärräkään.

        Mutta näinhän se menee, elämä on evokeille selviytymistaistelua jossa kaikki keinot ovat sallittuja ja varsinkaan moraalia ei tunneta.

        En vastaa trollikirjoituksiin muuten kuin huomauttamalla, että ne ovat trollaamista.


    • väsyneen vastaus

      "Elämä ei edellytä, että sillä olisi merkitys."

      Tämän näemme eri tavalla. Minusta elämän edellytys on juuri merkitys. Emme ehkä ymmärrä liiskatun muurahaisen elämän merkitystä, mutta ymmärtämättömyytemme ei tee sitä tyhjäksi, että jostain syystä juuri se muurahainen on syntynyt maailmaan ja juuri se tulee liiskatuksi. Jos puhutaan ainutkertaisesta ihmiselämästä, ajattelusta, tunteista, historiasta, telepatiasta, selittämättömistä signaaleista ihmisten välillä jne., täytyyhän kaikella olla jokin syvempi taustavoima, jota emme pysty järjellä selittämään. Ja jos emme pysty järkeistämään sitä, voimme saman tien myöntää, että on varmasti muutakin, mihin järkemme ei riitä. Tietoisuus on kyllä kiinteästi yhteydessä aivoihin, mutta mikä sen tietoisuuden ja persoonan luo sinne aivoihin ja miksi? Voidaan toki puhua kemiallisista yhdisteistä, hermosoluista, impulsseista jne., mutta mikä ne valmistaa ja saa toimimaan juuri tietyllä tavalla tietyssä yksilössä? Ei mikään synny tyhjiössä eikä vailla syytä syntymiseen.

      "on loukkaavaa että kreationismi esitetään tieteenä tai tieteen kaltaisena, koska sitä se ei ole"

      Myönnän, etten tunne kreationismia tarkemmin. Uskon sen olevan noin, kuten sanot. Se vain on niin, että taidatte pelata eri säännöillä. Olet oppinut ajattelemaan, että tieteellisen teorian taustalla on tutkimustuloksia, jotka on osoitettu luotettaviksi. Tulet ärtyneeksi, ettei jokin ryhmittymä pysty perustelemaan teorioitaan vaan esittää väitteitä, jotka tuntuvat tuulesta temmatuilta, ja kehtaa vielä väittää niitä tieteeksi. Se tuntuu sinusta epärehelliseltä, koska se on vastoin tieteen yleisiä periaatteita. Luulenpa, ettei kreationisteilla olekaan ensisijainen tavoite pyrkiä tieteellisesti validiin selitysmalliin vaan että he etsivät totuuttansa omilla säännöillään. Miksi se ärsyttää niin paljon?

      No, eipä ole minullakaan mitään sanottavaa tähän enkä jaksa muutenkaan perehtyä riittävän syvällisesti näihin juttuihin täällä. Kävin lueskelemassa unta odotellessani ja huvituin, miten eri tavalla kaksi osapuolta voi nähdä saman asian. Selvästikään mitään toisen osapuolen ymmärtämistä ei ole odotettavissa, kun lähtökohtanne ovat noin erilaiset. Hyvää yötä.

      • Totuudentorvettaja

        "Tämän näemme eri tavalla. Minusta elämän edellytys on juuri merkitys."

        Mikä mekanismi edellyttäisi, että sillä pitäisi olla merkitys? Emme puhu nyt filosofisesti, vaan käytännön tasolla. Jos elämällä on oltava tarkoitus voidakseen olla olemassa, niin kivelläkin pitää olla syy olla hiekkarannalla, vai onko niin, että kivi nyt vain on hiekkarannalla ilman sen suurempaa syytä.

        "telepatiasta, selittämättömistä signaaleista ihmisten välillä"

        Ei milloinkaan todistettu olemassaolevaksi. Päinvastaista näyttöä sen sijaan löytyy.

        "...täytyyhän kaikella olla jokin syvempi taustavoima, jota emme pysty järjellä selittämään."

        Ei mitään syytä, miksi pitäisi olla. Jos keksit jonkin syyn, niin kerro ihmeessä.

        "Tietoisuus on kyllä kiinteästi yhteydessä aivoihin, mutta mikä sen tietoisuuden ja persoonan luo sinne aivoihin"

        Aivosolujen lukemattomat vuorovaikutukset. Samasta syystä niihin voi myös vaikuttaa vaikuttamalla näihin vuorovaikutuksiin. Aivot toimivat toisin kuin tietokone - tietokoneen muistissa oleva tieto ei vaikuta muistin rakenteeseen, kun taas aivoissa muistijäljet luovat uusia yhteyksiä.

        ".. ja miksi?"

        Merkityksetön kysymys kun ymmärtää ettei tietoisuus ole tullut mistään ulkoapäin.

        "Olet oppinut ajattelemaan, että tieteellisen teorian taustalla on tutkimustuloksia, jotka on osoitettu luotettaviksi."

        Jos viittaat evoluutioteoriaan, niin sen tueksi on valtava määrä näyttöä useilta eri tieteenaloilta. Eri todistesuuntia on niin monta ja ne sopivat niin hyvin yhteen punoen todisteiden verkon, että teoriaa voidaan pitää hyvin luotettavana. Kaikki lehdistön tieteeksi tituleeraama lööppimateriaali ei ole oikeaa tiedettä ja itse suhtaudun hyvin suurella varauksella sellaisiin tutkimuksiin, jotka perustuvat ihmisiltä tehtyihin kyselyihin. Sen sijaan ajoitusmenetelmät, fossiililöydöt, dna:sta nähtävät asiat ja sellaiset ovat kokonaisuutena hyvin luotettavia vaikka kokonaisuudesta muutama harha-askel löytyisikin.

        "Luulenpa, ettei kreationisteilla olekaan ensisijainen tavoite pyrkiä tieteellisesti validiin selitysmalliin vaan että he etsivät totuuttansa omilla säännöillään. Miksi se ärsyttää niin paljon?"

        Kreationistit ovat pyrkineet kautta vuosien saada oppiaan kouluihin biologian tunneille tieteenä, ovat kirjoittaneet lukemattomia tekstejä joissa väitetään että evoluutioteoria ei ole tiedettä, työnnetään sanoja oikeiden tiedemiesten suihin editoimalla teksteistä sanoja pois lauseiden keskeltä tai katkaisemalla lauseita kesken, esittämällä kuulijoille että evoluutioteoria olisi kaatumassa tai että sen kannatus olisi tiedemaailmassa/tutkijoiden kesken romahtamassa ja niin edelleen.

        Kreationismi ärsyttää, koska kreationistit a) valehtelevat usein ja toistuvasti b) vääristelevät toisten sanoja, kirjoituksia ja tutkimustuloksia omaksi edukseen c) mustamaalaavat evoluutioteoriaa vääristelemällä mitä se tarkoittaa (teoria ei esimerkiksi väitä että kissa synnyttäisi koiran, että apina olisi synnyttänyt ihmisen tai että kivestä syntyisi elämää) d) yrittävät esittää kreationismia tieteenä - uskot tai et.

        Yötä


    • Aamukahvia, ole hyvä

      Tunnen itseni epävarmaksi kirjoittaessani asiasta, josta en tiedä riittävästi. Tarkoitan tällä kreationismia (ja miksen myös evoluutioteoriaa). Jos kreationistit ja evolutistit asettuvat janan ääripäihin, sijoitun itse ilmeisesti keskivälille. Uskon jossakin määrin evoluutioteoriaan, koska - kuten mainitsit - on olemassa monia todisteita siitä ja koska voin nähdä omankin elämäni aikana tapahtuneen monenlaista kehitystä yhteiskunnassa ja sen myötä myös ihmisten osaamisessa, asenteissa, tavoissa jne. Eläimiä ja kasveja on tänäkin aikana kuollut sukupuuttoon ja uusia on löydetty jne.

      En kuitenkaan näe elämän syntyä niin, etteikö luomiskertomus olisi mahdollinen. Itse asiassa uskon sen olevan kehysteoria, jonka sisälle evoluutioteoria sijoittuu. Uskon Jumalan olevan kaiken elämän luoja ja ylläpitäjä ja että evoluutio ja kehittyminen ovat hänen tapansa luoda ja jatkaa elämää.

      "Ei milloinkaan todistettu olemassaolevaksi. Päinvastaista näyttöä sen sijaan löytyy."

      Ei niin, koska järjen ulkopuolisia havaintoja ei voi tietenkään järjellä selittää. Tiedehän perustuu vahvasti logiikkaan eli järkeen, mutta on olemassa myös syvempi tietoisuus, jota ei järjellä ei selitetä. Siihen saa kosketuksen aisteilla ja tunteilla. Eikö sinulla ole kokemusta telepatiasta tai tunteesta, että ymmärrät jotakuta ihmistä hyvin ilman, että todellisuudessa tunnet häntä? Etkö tunne koskaan empatiaa lähes tuntematonta ihmistä kohtaan? Onko puhelimesi soinut joskus niin, että olet juuri ajatellut soittajaa? Osaatko toisinaan ennakoida tai aavistaa tulevia tapahtumia tai vaistota toisten ihmisten välisiä kemioita? Kaikkea ei tarvitse todistaa tieteellisin keinoin, vaan omatkin kokemuksesi ovat riittävä todiste siitä, että on olemassa jokin suurempi voima ympärillämme.

      "Ei mitään syytä, miksi pitäisi olla. Jos keksit jonkin syyn, niin kerro ihmeessä."

      En osaa kertoa enkä tiedä. Meistä kukaan ei tiedä mutta tuntee sen itsessään kyllä, jos on itselleen rehellinen. Jos olisit varma asiastasi, et kokisi tarvetta keskustella mieltäsi askarruttavista asioista uskovien kanssa. Et hakisi vastauksia kysymyksiin, jotka nousevat joltakin tietoisuutesi tasolta mieleesi. Myöntänet itsellesi, että vaikka paljon pystyt Jumalan sinulle antamalla viisaudella ymmärtämään, sinulla on paljon ajatuksia, toiveita, haluja, kiinnostuksenkohteita ja kysymyksiä, joita et voi järjellä hallita.

      Kreationistien toimintatavat eivät kuulosta oikein reiluilta, mikäli asia on, kuten kuvailet. Mutta antanet heille anteeksi ymmärtäessäsi, että he uskovat vakaasti asiaansa ja ovat vakuuttuneita sen oikeudellisuudesta. He haluavat vilpittömästi muidenkin ymmärtävän heidän löytämänsä totuuden. Oikeastaanhan teillä evoilla on aivan sama päämäärä, vaikka lähestytte asiaa vastakkaisesta näkökulmasta.

      En jaksa jatkaa tätä keskustelua. Se on omalta puoleltani hyödytöntä, sillä länkytän aivan turhaa ja tyhjää tänne, koska en tiedä aiheesta riittävästi ja koska en osaa sen vuoksi tuoda uusia näkökulmia asioihin.

    • Totuudentorvettaja

      "Ei niin, koska järjen ulkopuolisia havaintoja ei voi tietenkään järjellä selittää. "

      En jaksa/ehdi vastata tekstiin kokonaisuudessaan, mutta tästä kommentoin sen verran, että vaikka telepatiaa tms. ei voisi järjellä selittää, sitä voi kuitenkin tutkia selvittämällä, onko ilmiö olemassa. Olemassaolevan tiedon perusteella telepatiaa ei ole havaittu. Asiaa ei muuta se, ymmärretäänkö ilmiötä vai ei. Tutkimiseksi riittää se, että tutkitaan kykeneekö joku telepatiaan tai onko telepatiasta havaintoja ja vastaus on siis ei.

    • Aloittajan kysymykseen on vastatta jo monta kertaa, mutta vastataan taas kerran.

      Kaikki luonnossa havaittavat eläimet ovat valmiiksi luotuja lopulliseen muotoonsa. Yhdessäkään ei näy mitään merkkejä, että ne olisivat jollain tavoin keskeneräisiä tai jossain kehitysvaiheessa joksikin toiseksi lajiksi.

      Kaikki fossiiliaineistosta löydetyt eläimet, sukupuuttoon kuolleita lukuun ottamatta, ovat aivan samanlaisia kuin nykyäänkin. Niissä ei ole tapahtunut mitään kehitystä, lukuun ottamatta lajinsisäistä rajallista muuntelua, jota aina tapahtuu luonnossa.

      Luonnossa emme voi havaita ainuttakaan todistetta evoluutiosta, joten ainoaksi selitykseksi jää, että Jumala on luonut kaikki eläimet täydellisiksi ja lopulliseen muotoonsa.

      • Totuudentorvettaja

        "Kaikki luonnossa havaittavat eläimet ovat valmiiksi luotuja lopulliseen muotoonsa. Yhdessäkään ei näy mitään merkkejä, että ne olisivat jollain tavoin keskeneräisiä tai jossain kehitysvaiheessa joksikin toiseksi lajiksi."

        Olet ymmärtänyt "välimuodon" idean väärin. Ei ole tarkoituskaan, että jokin olisi puoliksi valmis ja pitää muistaa, että takana on miljardien vuosien kehitys. Muutos ei tarkoita muuttumista keskeneräisestä valmiiksi. Toisaalta esimerkiksi ihmistä voisi pitää keskeneräisenä, koska selkämme ei ole vielä täysin sopeutunut pystykävelyyn, silmiämme ei ole suunniteltu lähellä olevien asioiden katsomiseen tuntikausia päivässä(lukeminen) ja niin edelleen.

        "Kaikki fossiiliaineistosta löydetyt eläimet, sukupuuttoon kuolleita lukuun ottamatta, ovat aivan samanlaisia kuin nykyäänkin. Niissä ei ole tapahtunut mitään kehitystä, lukuun ottamatta lajinsisäistä rajallista muuntelua, jota aina tapahtuu luonnossa."

        No tämähän on aivan täyttä roskaa ja tiedät sen itsekin. Voit toki uskotella itsellesi mitä haluat, mutta totuus on aivan toinen.

        "Luonnossa emme voi havaita ainuttakaan todistetta evoluutiosta"

        Kehälajit ovat hyvä esimerkki, sekä muut lajiutumiset (laji muuttuu niin erilaikseksi, ettei kykene enää synnyttämään lisääntymiskelpoisia jälkeläisiä). Fossiililöydöksistäkin muutos nähdään varsin hyvin.

        Intä vaan Jaakob, mutta eivät tosiasiat muuksi muutu


      • Totuudentorvettaja kirjoitti:

        "Kaikki luonnossa havaittavat eläimet ovat valmiiksi luotuja lopulliseen muotoonsa. Yhdessäkään ei näy mitään merkkejä, että ne olisivat jollain tavoin keskeneräisiä tai jossain kehitysvaiheessa joksikin toiseksi lajiksi."

        Olet ymmärtänyt "välimuodon" idean väärin. Ei ole tarkoituskaan, että jokin olisi puoliksi valmis ja pitää muistaa, että takana on miljardien vuosien kehitys. Muutos ei tarkoita muuttumista keskeneräisestä valmiiksi. Toisaalta esimerkiksi ihmistä voisi pitää keskeneräisenä, koska selkämme ei ole vielä täysin sopeutunut pystykävelyyn, silmiämme ei ole suunniteltu lähellä olevien asioiden katsomiseen tuntikausia päivässä(lukeminen) ja niin edelleen.

        "Kaikki fossiiliaineistosta löydetyt eläimet, sukupuuttoon kuolleita lukuun ottamatta, ovat aivan samanlaisia kuin nykyäänkin. Niissä ei ole tapahtunut mitään kehitystä, lukuun ottamatta lajinsisäistä rajallista muuntelua, jota aina tapahtuu luonnossa."

        No tämähän on aivan täyttä roskaa ja tiedät sen itsekin. Voit toki uskotella itsellesi mitä haluat, mutta totuus on aivan toinen.

        "Luonnossa emme voi havaita ainuttakaan todistetta evoluutiosta"

        Kehälajit ovat hyvä esimerkki, sekä muut lajiutumiset (laji muuttuu niin erilaikseksi, ettei kykene enää synnyttämään lisääntymiskelpoisia jälkeläisiä). Fossiililöydöksistäkin muutos nähdään varsin hyvin.

        Intä vaan Jaakob, mutta eivät tosiasiat muuksi muutu

        xxxxOlet ymmärtänyt "välimuodon" idean väärin. Ei ole tarkoituskaan, että jokin olisi puoliksi valmis ja pitää muistaa, että takana on miljardien vuosien kehitys. Muutos ei tarkoita muuttumista keskeneräisestä valmiiksi.xxxxxx

        Välimuoto on aina puoliksi tai muuten osaksi valmis. Esim. jos lisko "kehittyy" nisäkkääksi, niin välimuodot ovat puoliksi tai osaksi valmiita nisäkkäitä ja sellaisia ei luonnosta ole tavattu. Väite miljardeista vuosista on täysin tekaistu ja tarkoitettu tukemaan evoluutioteoriaan. Sitä paitsi kukaan ei ole miljardeja vuosia sitten ollut havainnomassa yhtään mitään. Väite ei siis perustu luonnossa tehtyihin havaintoihin.


        xxxxToisaalta esimerkiksi ihmistä voisi pitää keskeneräisenä, koska selkämme ei ole vielä täysin sopeutunut pystykävelyyn, silmiämme ei ole suunniteltu lähellä olevien asioiden katsomiseen tuntikausia päivässä(lukeminen) ja niin edelleen. xxxx

        Selkärankamme on täysin suunniteltu pystyssä kävelemiseen. Ainakaan minulla ei ole mitään ongelmia selkärankani kanssa kun kävelen pystyssä.
        Silmät ovat myös suunniteltu lopulliseen muotoonsa. Niitä ei ole tarkoitettukaan käytettäväksi siten, että luetaan tuntikausia päivässä.

        ("Kaikki fossiiliaineistosta löydetyt eläimet, sukupuuttoon kuolleita lukuun ottamatta, ovat aivan samanlaisia kuin nykyäänkin. Niissä ei ole tapahtunut mitään kehitystä, lukuun ottamatta lajinsisäistä rajallista muuntelua, jota aina tapahtuu luonnossa.")

        xxxx No tämähän on aivan täyttä roskaa ja tiedät sen itsekin. Voit toki uskotella itsellesi mitä haluat, mutta totuus on aivan toinen. xxxxxx

        Fossiiliaineistosta ei ole löydetty yhtäkään välimuotoa. Taiteilijat ovat vain piirrelleet mielikuvituksensa pohjalta uskoteltuja välimuotoja muutamista luunsiruista, hampaista tai nikamista.

        ("Luonnossa emme voi havaita ainuttakaan todistetta evoluutiosta")

        xxxxx Kehälajit ovat hyvä esimerkki, sekä muut lajiutumiset (laji muuttuu niin erilaikseksi, ettei kykene enää synnyttämään lisääntymiskelpoisia jälkeläisiä). Fossiililöydöksistäkin muutos nähdään varsin hyvin.xxxxxx

        Kyse on pelkästä lajinsisäisestä muuntelusta eli ns. mikroevoluutiosta, eikä mistään evoluutiosta. Kyse on lajinsisäisestä muuntelusta siitäkin huolimatta, että osapuolet eivät pystyisi lisääntymään keskenään.

        Olen siis edelleen vahvasti sillä kannalla, että kaikki luonnossa tehdyt havainnot todistavat kreationismista ja evoluutiosta ei ole ainuttakaan todistetta.


      • Totuudentorvettaja
        Jaakob kirjoitti:

        xxxxOlet ymmärtänyt "välimuodon" idean väärin. Ei ole tarkoituskaan, että jokin olisi puoliksi valmis ja pitää muistaa, että takana on miljardien vuosien kehitys. Muutos ei tarkoita muuttumista keskeneräisestä valmiiksi.xxxxxx

        Välimuoto on aina puoliksi tai muuten osaksi valmis. Esim. jos lisko "kehittyy" nisäkkääksi, niin välimuodot ovat puoliksi tai osaksi valmiita nisäkkäitä ja sellaisia ei luonnosta ole tavattu. Väite miljardeista vuosista on täysin tekaistu ja tarkoitettu tukemaan evoluutioteoriaan. Sitä paitsi kukaan ei ole miljardeja vuosia sitten ollut havainnomassa yhtään mitään. Väite ei siis perustu luonnossa tehtyihin havaintoihin.


        xxxxToisaalta esimerkiksi ihmistä voisi pitää keskeneräisenä, koska selkämme ei ole vielä täysin sopeutunut pystykävelyyn, silmiämme ei ole suunniteltu lähellä olevien asioiden katsomiseen tuntikausia päivässä(lukeminen) ja niin edelleen. xxxx

        Selkärankamme on täysin suunniteltu pystyssä kävelemiseen. Ainakaan minulla ei ole mitään ongelmia selkärankani kanssa kun kävelen pystyssä.
        Silmät ovat myös suunniteltu lopulliseen muotoonsa. Niitä ei ole tarkoitettukaan käytettäväksi siten, että luetaan tuntikausia päivässä.

        ("Kaikki fossiiliaineistosta löydetyt eläimet, sukupuuttoon kuolleita lukuun ottamatta, ovat aivan samanlaisia kuin nykyäänkin. Niissä ei ole tapahtunut mitään kehitystä, lukuun ottamatta lajinsisäistä rajallista muuntelua, jota aina tapahtuu luonnossa.")

        xxxx No tämähän on aivan täyttä roskaa ja tiedät sen itsekin. Voit toki uskotella itsellesi mitä haluat, mutta totuus on aivan toinen. xxxxxx

        Fossiiliaineistosta ei ole löydetty yhtäkään välimuotoa. Taiteilijat ovat vain piirrelleet mielikuvituksensa pohjalta uskoteltuja välimuotoja muutamista luunsiruista, hampaista tai nikamista.

        ("Luonnossa emme voi havaita ainuttakaan todistetta evoluutiosta")

        xxxxx Kehälajit ovat hyvä esimerkki, sekä muut lajiutumiset (laji muuttuu niin erilaikseksi, ettei kykene enää synnyttämään lisääntymiskelpoisia jälkeläisiä). Fossiililöydöksistäkin muutos nähdään varsin hyvin.xxxxxx

        Kyse on pelkästä lajinsisäisestä muuntelusta eli ns. mikroevoluutiosta, eikä mistään evoluutiosta. Kyse on lajinsisäisestä muuntelusta siitäkin huolimatta, että osapuolet eivät pystyisi lisääntymään keskenään.

        Olen siis edelleen vahvasti sillä kannalla, että kaikki luonnossa tehdyt havainnot todistavat kreationismista ja evoluutiosta ei ole ainuttakaan todistetta.

        Intä vaan. Turha sinulle on edes vastata. Tiedät itsekin varsin hyvin, että löydetyt välimuodot eivät ole mitään taiteilijoiden tekeleitä, vaan maasta kaivettuja luita. Toki niistä on voitu hahmotella, miltä eläin olisi näyttänyt, mutta se on sivuseikka.


      • Jaakob kirjoitti:

        xxxxOlet ymmärtänyt "välimuodon" idean väärin. Ei ole tarkoituskaan, että jokin olisi puoliksi valmis ja pitää muistaa, että takana on miljardien vuosien kehitys. Muutos ei tarkoita muuttumista keskeneräisestä valmiiksi.xxxxxx

        Välimuoto on aina puoliksi tai muuten osaksi valmis. Esim. jos lisko "kehittyy" nisäkkääksi, niin välimuodot ovat puoliksi tai osaksi valmiita nisäkkäitä ja sellaisia ei luonnosta ole tavattu. Väite miljardeista vuosista on täysin tekaistu ja tarkoitettu tukemaan evoluutioteoriaan. Sitä paitsi kukaan ei ole miljardeja vuosia sitten ollut havainnomassa yhtään mitään. Väite ei siis perustu luonnossa tehtyihin havaintoihin.


        xxxxToisaalta esimerkiksi ihmistä voisi pitää keskeneräisenä, koska selkämme ei ole vielä täysin sopeutunut pystykävelyyn, silmiämme ei ole suunniteltu lähellä olevien asioiden katsomiseen tuntikausia päivässä(lukeminen) ja niin edelleen. xxxx

        Selkärankamme on täysin suunniteltu pystyssä kävelemiseen. Ainakaan minulla ei ole mitään ongelmia selkärankani kanssa kun kävelen pystyssä.
        Silmät ovat myös suunniteltu lopulliseen muotoonsa. Niitä ei ole tarkoitettukaan käytettäväksi siten, että luetaan tuntikausia päivässä.

        ("Kaikki fossiiliaineistosta löydetyt eläimet, sukupuuttoon kuolleita lukuun ottamatta, ovat aivan samanlaisia kuin nykyäänkin. Niissä ei ole tapahtunut mitään kehitystä, lukuun ottamatta lajinsisäistä rajallista muuntelua, jota aina tapahtuu luonnossa.")

        xxxx No tämähän on aivan täyttä roskaa ja tiedät sen itsekin. Voit toki uskotella itsellesi mitä haluat, mutta totuus on aivan toinen. xxxxxx

        Fossiiliaineistosta ei ole löydetty yhtäkään välimuotoa. Taiteilijat ovat vain piirrelleet mielikuvituksensa pohjalta uskoteltuja välimuotoja muutamista luunsiruista, hampaista tai nikamista.

        ("Luonnossa emme voi havaita ainuttakaan todistetta evoluutiosta")

        xxxxx Kehälajit ovat hyvä esimerkki, sekä muut lajiutumiset (laji muuttuu niin erilaikseksi, ettei kykene enää synnyttämään lisääntymiskelpoisia jälkeläisiä). Fossiililöydöksistäkin muutos nähdään varsin hyvin.xxxxxx

        Kyse on pelkästä lajinsisäisestä muuntelusta eli ns. mikroevoluutiosta, eikä mistään evoluutiosta. Kyse on lajinsisäisestä muuntelusta siitäkin huolimatta, että osapuolet eivät pystyisi lisääntymään keskenään.

        Olen siis edelleen vahvasti sillä kannalla, että kaikki luonnossa tehdyt havainnot todistavat kreationismista ja evoluutiosta ei ole ainuttakaan todistetta.

        >Silmät ovat myös suunniteltu lopulliseen muotoonsa. Niitä ei ole tarkoitettukaan käytettäväksi siten, että luetaan tuntikausia päivässä.

        Niin, mistäpä jumala olisi voinut aavistaa, että jonakin päivänä sadat miljoonat ihmiset lukevat ihan hemmetisti.

        Eikä yksikään profetia osannut kertoa tästäkään todellisen tulevaisuuden asiasta tuon taivaallista, vaikka primitiiviset verikauhukuvat olivat niissä kyllä hyvin edustettuina.


      • RepeRuutikallo kirjoitti:

        >Silmät ovat myös suunniteltu lopulliseen muotoonsa. Niitä ei ole tarkoitettukaan käytettäväksi siten, että luetaan tuntikausia päivässä.

        Niin, mistäpä jumala olisi voinut aavistaa, että jonakin päivänä sadat miljoonat ihmiset lukevat ihan hemmetisti.

        Eikä yksikään profetia osannut kertoa tästäkään todellisen tulevaisuuden asiasta tuon taivaallista, vaikka primitiiviset verikauhukuvat olivat niissä kyllä hyvin edustettuina.

        xxxx Niin, mistäpä jumala olisi voinut aavistaa, että jonakin päivänä sadat miljoonat ihmiset lukevat ihan hemmetisti.xxxxxx

        Jumala ei luonut silmiä toimimaan ihmisen ehdoilla.

        xxxxx Eikä yksikään profetia osannut kertoa tästäkään todellisen tulevaisuuden asiasta tuon taivaallista, vaikka primitiiviset verikauhukuvat olivat niissä kyllä hyvin edustettuina.xxxxxx

        Ne, jotka uskovat Jumalaan, eivät tarvitse sellaista profetiaa, ja ne jotka ovat ateisteja, eivät usko, vaikka sellainen profetia olisikin. Ei siis ole ketään, jollle kyseinen profetia olisi tarpeellinen.


      • Jaakob kirjoitti:

        xxxx Niin, mistäpä jumala olisi voinut aavistaa, että jonakin päivänä sadat miljoonat ihmiset lukevat ihan hemmetisti.xxxxxx

        Jumala ei luonut silmiä toimimaan ihmisen ehdoilla.

        xxxxx Eikä yksikään profetia osannut kertoa tästäkään todellisen tulevaisuuden asiasta tuon taivaallista, vaikka primitiiviset verikauhukuvat olivat niissä kyllä hyvin edustettuina.xxxxxx

        Ne, jotka uskovat Jumalaan, eivät tarvitse sellaista profetiaa, ja ne jotka ovat ateisteja, eivät usko, vaikka sellainen profetia olisikin. Ei siis ole ketään, jollle kyseinen profetia olisi tarpeellinen.

        >Ne, jotka uskovat Jumalaan, eivät tarvitse sellaista profetiaa, ja ne jotka ovat ateisteja, eivät usko, vaikka sellainen profetia olisikin. Ei siis ole ketään, jollle kyseinen profetia olisi tarpeellinen.

        Suurin osa ihmisistä ei kuulu kumpaankaan järkyttävän mustavalkoisen maailmasi luokkaan.

        Vai onko mielestäsi niin, että kun uskovaisten tutkimusten mukaan 2 % ihmisistä on ateisteja, loput 98 % uskovat juuri sinun Jumalaasi?


      • Totuudentorvettaja
        Jaakob kirjoitti:

        xxxx Niin, mistäpä jumala olisi voinut aavistaa, että jonakin päivänä sadat miljoonat ihmiset lukevat ihan hemmetisti.xxxxxx

        Jumala ei luonut silmiä toimimaan ihmisen ehdoilla.

        xxxxx Eikä yksikään profetia osannut kertoa tästäkään todellisen tulevaisuuden asiasta tuon taivaallista, vaikka primitiiviset verikauhukuvat olivat niissä kyllä hyvin edustettuina.xxxxxx

        Ne, jotka uskovat Jumalaan, eivät tarvitse sellaista profetiaa, ja ne jotka ovat ateisteja, eivät usko, vaikka sellainen profetia olisikin. Ei siis ole ketään, jollle kyseinen profetia olisi tarpeellinen.

        "xxxx Niin, mistäpä jumala olisi voinut aavistaa, että jonakin päivänä sadat miljoonat ihmiset lukevat ihan hemmetisti.xxxxxx

        Jumala ei luonut silmiä toimimaan ihmisen ehdoilla."

        Siksi Raamattukin julkaistiin alunperin äänikirjana?


    • jyrtsipoi

      Aineen synty.

      Elämän synty.

      Ajattelun synty.

      Ihana maapallo.

      Ihana minä, siis se että saan elää ja nähdä paljon hyvää.

      Ihanampi elinikäinen vaimoni, mummo jo mutta maku vain paranee kuin hyvässä vuosikertaviinissä.

      Ihanat lapset.

      Ihanat lapsenlapset.

      Ihana Kreikan saaristo.

      Ihana Krabi.

      Ihana terve henkisesti onnellinen elämä via Biblia.

      Ihanaa että nykyinen elämämme ei ole kaikki.

      Riittääkö nämä?

    • muutama

      Kreationismin todisteita

      1. informaatiota tuhoavia mutaatioita havaittu runsaasti
      2. aidosti hyödyllisiä mutaatioita ei ole havaittu ollenkaan
      3. surkastuneet elimet
      4. perinnölliset sairaudet
      5. luonnonvalinta todistettu
      6. lajiutuminen osapopulaatioihin
      7. maapallon magn kenttä (kok. energia) heikkenee
      8. menneisyydessä tapahtuneet nopeat magn kentän muutokset tallentuneet kallioperään
      9. "ylimääräinen" C14
      10. helium zirkoneissa
      11. lyijy zirkoneissa
      12. galaksien sisäiset kiertonopeudet "liian" suuria
      13. komeetat hajoavat "liian" nopeasti
      14. mantereet ja sedimentit edelleenkin olemassa nopeasta eroosiosta huolimatta
      15. merenpohjan kerrostumat "liian" ohuita
      16. meriveden suolapitoisuus "liian" alhainen (kertymän ja poistuman epätasapaino)
      17. evolutionistien kykenemättömyys puolustaa teoriaansa/kritisoida kreationismia avoimesti ja asiallisesti
      18. Raamatun luomiskertomus (kirjallinen todiste)
      19. eri kansoilla muita luomiskertomuksia
      20. Po-halot
      21. fissiourat
      22. kirjoitettu historia vain n. 5000 vuotta
      23. Haldanen dilemma
      24. Pioneer-anomalia (kreationistinen White Hole-kosmologia selittää, evolutionistinen Big Bang-kosmologia ei)
      25. pimeää ainetta ja pimeää energiaa "liian" vähän
      26. Linnunradan keskustassa kirkkaiden tähtien klustereita
      27. keskustaa kiertää kaasukehä
      28. keskustassa erittäin kuumaa röntgensäteilevää kaasua
      29. Kuun loittoneminen
      30. polven monimutkaisuus
      31. riikinkukon pyrstön kauneus (seksuaalivalinta ei selitä)
      32. proteiinien laskostuminen
      33. termodynamiikan 2. pääsääntö
      34. Dembskin matemaattinen todistus
      35. Laonastes aenigmamus, Neoglyphea neocaledonicaksi, Latimeria chalumnae, jne
      36. Mammutin DNA:n säilyminen
      37. Tyrannosauruksen luista löydetty punasoluja
      38. Neptunuksen ja Uranuksen magneettikentät
      39. heterophyid trematode -loismadot
      40. Kingston peak -muodostelma
      41. Heikon Auringon paradoksi
      42. Merkurius (nopea synty, ominaisuudet)
      43. ATP
      44. B-solut
      45. Keuhkoton sammakko
      46. Nanotimanttien puuttuminen
      47. Kielen alkuperä (evolutionistien teoriat ei selitä)
      48. Uox-pseudogeeni, GULO-pseudogeeni
      49. Three sisters -muodostelma
      50. Vedyn ja heliumin määrä maailmankaikkeudessa


      http://personal.inet.fi/koti/tjt/evkreatodisteet.html

      • Totuudentorvettaja

        Eivät nuo ole kreationismia tukevia todisteita.

        Esim:

        "5. luonnonvalinta todistettu"
        "6. lajiutuminen osapopulaatioihin"

        Sitten on jotain todella päättömiä. Esim.

        "17. evolutionistien kykenemättömyys puolustaa teoriaansa/kritisoida kreationismia avoimesti ja asiallisesti"

        Kuinka paljon avoimemmin niitä voi kritisoida ja vastustaa? Lisäksi tiedepuoli sentään perustee tosiasiapohjaisesti antamansa kritiikin.

        "18. Raamatun luomiskertomus (kirjallinen todiste)"

        Kuten Koraani, Kalevala...

        "19. eri kansoilla muita luomiskertomuksia"

        Niinpä...

        Sitten on näitä aikoja sitten kaatuneita olkiukkoja, joita kreationistitkaan eivät yleensä enää käytä. Esim.

        "33. termodynamiikan 2. pääsääntö" joka on suorastaan klassikkoargumentti kreationisteilta ja jonka lapsikin osaa selittää kumoon

        "29. Kuun loittoneminen" Ja jos ulkona sataa sentin vettä, niin vuodessa maa on 365 cm veden alla? ;)

        Ota listastasi pari parasta vaihtoehtoa ja kirjoita, miten ne todistavat kreationismin puolesta. Tuollaisen listan käsitteleminen kohta kohdalta on liian työlästä, mutta mitä sitä selasin, se tuntui olevan täynnä moneen kertaan kumottuja juttuja. Ota muutama paras esimerkki ja selitä niiden merkitys kreationismille, niin katsotaan tarkemmin.


      • Totuudentorvettaja kirjoitti:

        Eivät nuo ole kreationismia tukevia todisteita.

        Esim:

        "5. luonnonvalinta todistettu"
        "6. lajiutuminen osapopulaatioihin"

        Sitten on jotain todella päättömiä. Esim.

        "17. evolutionistien kykenemättömyys puolustaa teoriaansa/kritisoida kreationismia avoimesti ja asiallisesti"

        Kuinka paljon avoimemmin niitä voi kritisoida ja vastustaa? Lisäksi tiedepuoli sentään perustee tosiasiapohjaisesti antamansa kritiikin.

        "18. Raamatun luomiskertomus (kirjallinen todiste)"

        Kuten Koraani, Kalevala...

        "19. eri kansoilla muita luomiskertomuksia"

        Niinpä...

        Sitten on näitä aikoja sitten kaatuneita olkiukkoja, joita kreationistitkaan eivät yleensä enää käytä. Esim.

        "33. termodynamiikan 2. pääsääntö" joka on suorastaan klassikkoargumentti kreationisteilta ja jonka lapsikin osaa selittää kumoon

        "29. Kuun loittoneminen" Ja jos ulkona sataa sentin vettä, niin vuodessa maa on 365 cm veden alla? ;)

        Ota listastasi pari parasta vaihtoehtoa ja kirjoita, miten ne todistavat kreationismin puolesta. Tuollaisen listan käsitteleminen kohta kohdalta on liian työlästä, mutta mitä sitä selasin, se tuntui olevan täynnä moneen kertaan kumottuja juttuja. Ota muutama paras esimerkki ja selitä niiden merkitys kreationismille, niin katsotaan tarkemmin.

        >Ota muutama paras esimerkki ja selitä niiden merkitys kreationismille, niin katsotaan tarkemmin.

        Eipä taida neiti papukaijalta onnistua. Tuskin itseään ymmärsi listaansa.



    • xxxxx Kolme-neljä viikkoa on ollut kreationisteillä aikaa listata kreationismia tukevia faktoja (joita Toni T lupaili) tai näyttöä kreationismin tueksi. Ainoa vaatimus on se, että pitää olla sellainen asia, jota tiede ei ole vielä kumonnut ja pitää perustua tutkimukselliseen näyttöön xxxxxx

      Olet jo asettanut ennakkoehdot, jonka mukaan todisteen pitää perustua tutkimukselliseen näyttöön. Näillä näytöillä evolutionistit tarkoittavat vain sellaisia näyttöjä, jotka he itse tulkitsevat evolutionistisista ennakkoehdoista käsin.
      Jokainen näyttö, jonka evolutionistit väittävät todistavan evoluutiosta, voidaan yhtä hyvin tulkita todistavan kreationismista. Siksi tuon toisen näkökannan.

      Kaikkein selkein todiste kreationismista on Jumalan sana.
      Sen perusteella tiedämme 100 %:n varmasti, että Jumala on luonut kaikki kasvit, eläimet ja ihmisen täysin valmiiksi ja lopulliseen muotoonsa ja lisääntymään lajinsa mukaan. Tämä ei sulje pois sitä, että lajeissa voi tapahtua rajallista, lajinsisäistä muuntelua, jota evolutionistit virheellisesti pitävät todisteena evoluutiosta.

      Kaikki evoluutiosta esitetyt todisteet, jotka ovat ristiriidassa Raamatun kanssa, ovat vääriä, olipa niiden esittäjät kuinka oppineita tiedemiehiä tahansa ja todisteet kuinka "vakuuttavia" tahansa. Tiedemiehet ja tutkijat ovat erehtyväisiä tulkinnoissaan ja epätäydellisiä tiedoissaan ja heidän lausuntojaan ohjaa heidän oma elämänkatsomuksensa, aatemaailmansa ja tiedeyhteisöjen painostus jne

      Sen sijaan Raamattu on Jumalan sanaa, erehtymätön ja vapaa kaikista niistä asioista, jotka sitovat tiedemiesten näkemyksiä. Raamattu on puolueeton ja absoluuttinen totuus. Siksi emme tarvitse erehtyväisten tutkijoiden "todisteita" sellaisista asioista, joista Raamattu on jo kertonut totuuden.

      • Ei onnistunut taaskaan. Kreationistit leveilevät usein sillä, että nimenomaan tieteelliset todisteet puoltavat heidän juttujaan. Tieteellisiä todisteita pyydettiin, eikä niitä saatu.

        Saatiin Jaakob taas nojaamaan tuohon kreationismille keskeiseen todistusaineistoon, eli jumalolennon sanaan. Lainaan Kekkosta:

        "Saatanan tunarit!"


    • Totuudentorvettaja

      "Jokainen näyttö, jonka evolutionistit väittävät todistavan evoluutiosta, voidaan yhtä hyvin tulkita todistavan kreationismista. Siksi tuon toisen näkökannan."

      Jos kreationismiin kuuluu nykyään lajien kehittyminen pitkällä aikavälillä, uusien lajien muodostuminen ja niin edelleen, niin sitten asia voi hyvinkin olla näin. Tosin tämä tarkoittaa että kreationistit ovat hyväksyneet kirkkojen näkemyksen evoluutiosta jumalan työkaluna.

      "Sen perusteella tiedämme 100 %:n varmasti, että Jumala on luonut kaikki kasvit, eläimet ja ihmisen täysin valmiiksi ja lopulliseen muotoonsa ja lisääntymään lajinsa mukaan."

      Tämä kirjoituksesi kumoaa tuon ylemmän tekstisi. Voisitko päättää, mitä mieltä olet. Tosiasia on, ettei vanhoista fossiililöydöistä löydy yhtään nykylajia, eikä nykyään elä yhtään todella vanhaa lajia - melko vanhoja tosin on joitakin, kuten krokotiilit. Se ettei "ennenmuinoin" elänyt yhtään nykypäivän lajeja on jo yksistäänkin aika tiukka todiste sen puolesta, että lajit ovat muuttuneet matkan varrella. Sinulla on ollut melkein kuukausi aikaa linkittää tänne esimerkki jostain fossiloituneesta nykyeläimestä, mutta sellaista ei ole näkynyt. Etkö jo vihdoin voisi myöntää kiveä kiveksi - ei se siitä inttämällä muutu.

      • xxxxx Jos kreationismiin kuuluu nykyään lajien kehittyminen pitkällä aikavälillä, uusien lajien muodostuminen ja niin edelleen, niin sitten asia voi hyvinkin olla näin. Tosin tämä tarkoittaa että kreationistit ovat hyväksyneet kirkkojen näkemyksen evoluutiosta jumalan työkaluna.xxxxxxx

        Kysymyshän oli todisteista. Lajien kehittymisestä pitkällä aikavälillä ei ole todisteita.
        ("Sen perusteella tiedämme 100 %:n varmasti, että Jumala on luonut kaikki kasvit, eläimet ja ihmisen täysin valmiiksi ja lopulliseen muotoonsa ja lisääntymään lajinsa mukaan.")

        xxxxx Tämä kirjoituksesi kumoaa tuon ylemmän tekstisi. Voisitko päättää, mitä mieltä olet. Tosiasia on, ettei vanhoista fossiililöydöistä löydy yhtään nykylajia, eikä nykyään elä yhtään todella vanhaa lajia - melko vanhoja tosin on joitakin, kuten krokotiilit. Se ettei "ennenmuinoin" elänyt yhtään nykypäivän lajeja on jo yksistäänkin aika tiukka todiste sen puolesta, että lajit ovat muuttuneet matkan varrella. Sinulla on ollut melkein kuukausi aikaa linkittää tänne esimerkki jostain fossiloituneesta nykyeläimestä, mutta sellaista ei ole näkynyt. Etkö jo vihdoin voisi myöntää kiveä kiveksi - ei se siitä inttämällä muutu xxxxxxx

        85 % nykyisin elävistä nisäkäslajeista on löydetty myös fossiileina ja ne ovat samanlaisia kuin nykyäänkin. Mitään evolutionistista kehitystä ei ole tapahtunut.


      • Jaakob kirjoitti:

        xxxxx Jos kreationismiin kuuluu nykyään lajien kehittyminen pitkällä aikavälillä, uusien lajien muodostuminen ja niin edelleen, niin sitten asia voi hyvinkin olla näin. Tosin tämä tarkoittaa että kreationistit ovat hyväksyneet kirkkojen näkemyksen evoluutiosta jumalan työkaluna.xxxxxxx

        Kysymyshän oli todisteista. Lajien kehittymisestä pitkällä aikavälillä ei ole todisteita.
        ("Sen perusteella tiedämme 100 %:n varmasti, että Jumala on luonut kaikki kasvit, eläimet ja ihmisen täysin valmiiksi ja lopulliseen muotoonsa ja lisääntymään lajinsa mukaan.")

        xxxxx Tämä kirjoituksesi kumoaa tuon ylemmän tekstisi. Voisitko päättää, mitä mieltä olet. Tosiasia on, ettei vanhoista fossiililöydöistä löydy yhtään nykylajia, eikä nykyään elä yhtään todella vanhaa lajia - melko vanhoja tosin on joitakin, kuten krokotiilit. Se ettei "ennenmuinoin" elänyt yhtään nykypäivän lajeja on jo yksistäänkin aika tiukka todiste sen puolesta, että lajit ovat muuttuneet matkan varrella. Sinulla on ollut melkein kuukausi aikaa linkittää tänne esimerkki jostain fossiloituneesta nykyeläimestä, mutta sellaista ei ole näkynyt. Etkö jo vihdoin voisi myöntää kiveä kiveksi - ei se siitä inttämällä muutu xxxxxxx

        85 % nykyisin elävistä nisäkäslajeista on löydetty myös fossiileina ja ne ovat samanlaisia kuin nykyäänkin. Mitään evolutionistista kehitystä ei ole tapahtunut.

        >85 % nykyisin elävistä nisäkäslajeista on löydetty myös fossiileina ja ne ovat samanlaisia kuin nykyäänkin.

        Niin varmaan kaatumaseuroissa, mutta tiede ei näitä fossiileja tunne. Etkä näköjään tunne sinäkään, kun mitään näyttöä ei löydy,


    • Totuudentorvettaja

      Jaakob: "Jokainen näyttö, jonka evolutionistit väittävät todistavan evoluutiosta, voidaan yhtä hyvin tulkita todistavan kreationismista. Siksi tuon toisen näkökannan."

      TT:n-vastaus: "Jos kreationismiin kuuluu nykyään lajien kehittyminen pitkällä aikavälillä, uusien lajien muodostuminen ja niin edelleen, niin sitten asia voi hyvinkin olla näin. Tosin tämä tarkoittaa että kreationistit ovat hyväksyneet kirkkojen näkemyksen evoluutiosta jumalan työkaluna."

      Jaakobin jatkovastaus: "Kysymyshän oli todisteista. Lajien kehittymisestä pitkällä aikavälillä ei ole todisteita."

      Fossiililöydöt todistavat yksinomaan sen puolesta, että aiemmin eli eri lajit kuin nykyään. Tämä sopii täydellisesti yhteen evoluutioteorian kanssa, mutta ei niin mitenkään kreationismin kanssa. Fossiilit ovat todellisia, käsin kosketeltavia ja usein menetelmin ajoitettavissa olevia löydöksiä, jotka ympäri maapalloa löytöpaikasta ja tutkijaryhmästä riippumatta osoittavat aina saman asian - ennen eli eri lajit kuin nykypäivänä. Jos väität että tämä sopii yhteen edustamasi kreationismin kanssa olet sekaisin kuin hullu (huomatkaa, en halunnut pilkata syyttömiä seinäkelloja).

      En usko että sinä tietoisesti teet sitä mitä teet. Joskus ajattelin niin, mutta kaikkien näiden kertojen ja kaiken tämän ajan jälkeen uskon kyllä, että puhut omasta mielestäsi totta. Et hahmota ollenkaan sitä, miten kiertelet asiaa ja miten asiat eivät nyt kertakaikkiaan ole niinkuin uskot/koet niiden olevan.

      On aika palata maan pinnalle. Älä stressaa itseäsi näillä asioilla, vaan ota rauhallisesti. Et joudu helvettiin, eikä kuuma kivi putoa päähäsi, eikä mikään jumala - on hän olemassa tai ei - ole sinulle vihainen, vaikka hyväksyisit että maailma ei nykytietämyksen mukaan näytä siltä kuin Raamatun kirjaimellinen tulkinta antaa ymmärtää. Maailmalla tarkoitan tässä esimerkiksi Raamatusta tulkittua maailman ikää ja samalla tavalla tehtyä tulkintaa lajien synnystä. En ota mitään kantaa siihen, onko jotain yliluonnollista vai ei, mutta jos maailma näyttää joltakin, niin siihen on syy. Ehkä se syy on se, että maailma tosiaan on täysin naturalististen lakien luoma tai sitten toinen vaihtoehto on se, että mikä ikinä kaiken taustalla onkaan, halusi sen näyttävän tältä.

      • xxxx Fossiililöydöt todistavat yksinomaan sen puolesta, että aiemmin eli eri lajit kuin nykyään. Tämä sopii täydellisesti yhteen evoluutioteorian kanssa, mutta ei niin mitenkään kreationismin kanssa. xxxxxx

        Ainoastaan ne olivat eri lajeja, jotka ovat kuolleet sukupuuttoon. Se ei ole mitenkään ristiriidassa kreationismin kanssa.

        xxxxxx Fossiilit ovat todellisia, käsin kosketeltavia ja usein menetelmin ajoitettavissa olevia löydöksiä, jotka ympäri maapalloa löytöpaikasta ja tutkijaryhmästä riippumatta osoittavat aina saman asian - ennen eli eri lajit kuin nykypäivänä. Jos väität että tämä sopii yhteen edustamasi kreationismin kanssa olet sekaisin kuin hullu (huomatkaa, en halunnut pilkata syyttömiä seinäkelloja) xxxxx

        Kuten jo sanoin, kyse on sukupuuttoon kuolleista lajeista, mikä ei ole ristiriidassa kreationismin kanssa. Jumala on luonut sekä sukupuuttoon kuolleet että nykyisin elävät lajit..

        xxxxx En usko että sinä tietoisesti teet sitä mitä teet. Joskus ajattelin niin, mutta kaikkien näiden kertojen ja kaiken tämän ajan jälkeen uskon kyllä, että puhut omasta mielestäsi totta. Et hahmota ollenkaan sitä, miten kiertelet asiaa ja miten asiat eivät nyt kertakaikkiaan ole niinkuin uskot/koet niiden olevan. xxxxx

        Uskon siihen, että Jumalan sana on totuus. Jos sinun "tieteesi" ei siihen yhdy, niin vika on "tieteessäsi" ja sen tulkinnoissa.

        xxxxx On aika palata maan pinnalle. Älä stressaa itseäsi näillä asioilla, vaan ota rauhallisesti. Et joudu helvettiin, eikä kuuma kivi putoa päähäsi, eikä mikään jumala - on hän olemassa tai ei - ole sinulle vihainen, vaikka hyväksyisit että maailma ei nykytietämyksen mukaan näytä siltä kuin Raamatun kirjaimellinen tulkinta antaa ymmärtää. Maailmalla tarkoitan tässä esimerkiksi Raamatusta tulkittua maailman ikää ja samalla tavalla tehtyä tulkintaa lajien synnystä. En ota mitään kantaa siihen, onko jotain yliluonnollista vai ei, mutta jos maailma näyttää joltakin, niin siihen on syy. Ehkä se syy on se, että maailma tosiaan on täysin naturalististen lakien luoma tai sitten toinen vaihtoehto on se, että mikä ikinä kaiken taustalla onkaan, halusi sen näyttävän tältä xxxxxx

        Minusta maailma näyttää luodulta ja uskon että se näyttää useimmista muistakin ihmisistä luodulta. Jumalfobia ja tiedeyhteisöjen paine, tiedemiesten ja tutkijoiden urakehitys, ympäristön paine ja muut psykologiset tekijät saavat aikaan sen, että kaikki eivät uskalla tunnustaa, että maailma näyttää luodulta.

        Ja Raamatun perusteella tiedämme myös, että se on luotu. Evoluutio ei ole persoona. Sillä ei ole aivoja, älyä, viisautta, tietoa eikä taitoa luoda eikä kehittää yhtään mitään. Sillä ei ole viisautta luoda eikä kehittää ainuttakaan kemiallista reaktiota, joita evoluutioteoria pitää tukipilareinaan.
        Evoluutioon uskominen on siten kaiken järjen ja tieteen vastaista.


      • Totuudentorvettaja
        Jaakob kirjoitti:

        xxxx Fossiililöydöt todistavat yksinomaan sen puolesta, että aiemmin eli eri lajit kuin nykyään. Tämä sopii täydellisesti yhteen evoluutioteorian kanssa, mutta ei niin mitenkään kreationismin kanssa. xxxxxx

        Ainoastaan ne olivat eri lajeja, jotka ovat kuolleet sukupuuttoon. Se ei ole mitenkään ristiriidassa kreationismin kanssa.

        xxxxxx Fossiilit ovat todellisia, käsin kosketeltavia ja usein menetelmin ajoitettavissa olevia löydöksiä, jotka ympäri maapalloa löytöpaikasta ja tutkijaryhmästä riippumatta osoittavat aina saman asian - ennen eli eri lajit kuin nykypäivänä. Jos väität että tämä sopii yhteen edustamasi kreationismin kanssa olet sekaisin kuin hullu (huomatkaa, en halunnut pilkata syyttömiä seinäkelloja) xxxxx

        Kuten jo sanoin, kyse on sukupuuttoon kuolleista lajeista, mikä ei ole ristiriidassa kreationismin kanssa. Jumala on luonut sekä sukupuuttoon kuolleet että nykyisin elävät lajit..

        xxxxx En usko että sinä tietoisesti teet sitä mitä teet. Joskus ajattelin niin, mutta kaikkien näiden kertojen ja kaiken tämän ajan jälkeen uskon kyllä, että puhut omasta mielestäsi totta. Et hahmota ollenkaan sitä, miten kiertelet asiaa ja miten asiat eivät nyt kertakaikkiaan ole niinkuin uskot/koet niiden olevan. xxxxx

        Uskon siihen, että Jumalan sana on totuus. Jos sinun "tieteesi" ei siihen yhdy, niin vika on "tieteessäsi" ja sen tulkinnoissa.

        xxxxx On aika palata maan pinnalle. Älä stressaa itseäsi näillä asioilla, vaan ota rauhallisesti. Et joudu helvettiin, eikä kuuma kivi putoa päähäsi, eikä mikään jumala - on hän olemassa tai ei - ole sinulle vihainen, vaikka hyväksyisit että maailma ei nykytietämyksen mukaan näytä siltä kuin Raamatun kirjaimellinen tulkinta antaa ymmärtää. Maailmalla tarkoitan tässä esimerkiksi Raamatusta tulkittua maailman ikää ja samalla tavalla tehtyä tulkintaa lajien synnystä. En ota mitään kantaa siihen, onko jotain yliluonnollista vai ei, mutta jos maailma näyttää joltakin, niin siihen on syy. Ehkä se syy on se, että maailma tosiaan on täysin naturalististen lakien luoma tai sitten toinen vaihtoehto on se, että mikä ikinä kaiken taustalla onkaan, halusi sen näyttävän tältä xxxxxx

        Minusta maailma näyttää luodulta ja uskon että se näyttää useimmista muistakin ihmisistä luodulta. Jumalfobia ja tiedeyhteisöjen paine, tiedemiesten ja tutkijoiden urakehitys, ympäristön paine ja muut psykologiset tekijät saavat aikaan sen, että kaikki eivät uskalla tunnustaa, että maailma näyttää luodulta.

        Ja Raamatun perusteella tiedämme myös, että se on luotu. Evoluutio ei ole persoona. Sillä ei ole aivoja, älyä, viisautta, tietoa eikä taitoa luoda eikä kehittää yhtään mitään. Sillä ei ole viisautta luoda eikä kehittää ainuttakaan kemiallista reaktiota, joita evoluutioteoria pitää tukipilareinaan.
        Evoluutioon uskominen on siten kaiken järjen ja tieteen vastaista.

        Intä vaan. Ei se mitään muuta. Tyhjiä puheita ja olkiukkoja on kyllä nähty aiemminkin.


      • Jaakob kirjoitti:

        xxxx Fossiililöydöt todistavat yksinomaan sen puolesta, että aiemmin eli eri lajit kuin nykyään. Tämä sopii täydellisesti yhteen evoluutioteorian kanssa, mutta ei niin mitenkään kreationismin kanssa. xxxxxx

        Ainoastaan ne olivat eri lajeja, jotka ovat kuolleet sukupuuttoon. Se ei ole mitenkään ristiriidassa kreationismin kanssa.

        xxxxxx Fossiilit ovat todellisia, käsin kosketeltavia ja usein menetelmin ajoitettavissa olevia löydöksiä, jotka ympäri maapalloa löytöpaikasta ja tutkijaryhmästä riippumatta osoittavat aina saman asian - ennen eli eri lajit kuin nykypäivänä. Jos väität että tämä sopii yhteen edustamasi kreationismin kanssa olet sekaisin kuin hullu (huomatkaa, en halunnut pilkata syyttömiä seinäkelloja) xxxxx

        Kuten jo sanoin, kyse on sukupuuttoon kuolleista lajeista, mikä ei ole ristiriidassa kreationismin kanssa. Jumala on luonut sekä sukupuuttoon kuolleet että nykyisin elävät lajit..

        xxxxx En usko että sinä tietoisesti teet sitä mitä teet. Joskus ajattelin niin, mutta kaikkien näiden kertojen ja kaiken tämän ajan jälkeen uskon kyllä, että puhut omasta mielestäsi totta. Et hahmota ollenkaan sitä, miten kiertelet asiaa ja miten asiat eivät nyt kertakaikkiaan ole niinkuin uskot/koet niiden olevan. xxxxx

        Uskon siihen, että Jumalan sana on totuus. Jos sinun "tieteesi" ei siihen yhdy, niin vika on "tieteessäsi" ja sen tulkinnoissa.

        xxxxx On aika palata maan pinnalle. Älä stressaa itseäsi näillä asioilla, vaan ota rauhallisesti. Et joudu helvettiin, eikä kuuma kivi putoa päähäsi, eikä mikään jumala - on hän olemassa tai ei - ole sinulle vihainen, vaikka hyväksyisit että maailma ei nykytietämyksen mukaan näytä siltä kuin Raamatun kirjaimellinen tulkinta antaa ymmärtää. Maailmalla tarkoitan tässä esimerkiksi Raamatusta tulkittua maailman ikää ja samalla tavalla tehtyä tulkintaa lajien synnystä. En ota mitään kantaa siihen, onko jotain yliluonnollista vai ei, mutta jos maailma näyttää joltakin, niin siihen on syy. Ehkä se syy on se, että maailma tosiaan on täysin naturalististen lakien luoma tai sitten toinen vaihtoehto on se, että mikä ikinä kaiken taustalla onkaan, halusi sen näyttävän tältä xxxxxx

        Minusta maailma näyttää luodulta ja uskon että se näyttää useimmista muistakin ihmisistä luodulta. Jumalfobia ja tiedeyhteisöjen paine, tiedemiesten ja tutkijoiden urakehitys, ympäristön paine ja muut psykologiset tekijät saavat aikaan sen, että kaikki eivät uskalla tunnustaa, että maailma näyttää luodulta.

        Ja Raamatun perusteella tiedämme myös, että se on luotu. Evoluutio ei ole persoona. Sillä ei ole aivoja, älyä, viisautta, tietoa eikä taitoa luoda eikä kehittää yhtään mitään. Sillä ei ole viisautta luoda eikä kehittää ainuttakaan kemiallista reaktiota, joita evoluutioteoria pitää tukipilareinaan.
        Evoluutioon uskominen on siten kaiken järjen ja tieteen vastaista.

        >Uskon siihen, että Jumalan sana on totuus. Jos sinun "tieteesi" ei siihen yhdy, niin vika on "tieteessäsi" ja sen tulkinnoissa.

        Koska sinä uskot, tästä seuraa, että muiden omastasi poikkeavissa käsityksissä on siis jokin absoluuttinen vika?

        Ymmärsinkö nyt oikein? Tuohan on järkyttävää suuruudenhulluutta. Miten ylipäätään pärjäät tässä ihmisten maailmassa? Rajoittuuko siviilielämäsi yksinomaan samoin uskovien keskuuteen (siis siihen lahkoon, jota et monista pyynnöistä huolimatta suostu paljastamaan koska haluat itse itsellesi antamin valtuuksin edustaa kaikkia kristittyjä)?


    • ´´Uskon siihen, että Jumalan sana on totuus. Jos sinun "tieteesi" ei siihen yhdy, niin vika on "tieteessäsi" ja sen tulkinnoissa.´´

      Käytännössä yrityksesi kääntää asiat päälaelleen noin ei toimi. Tiede on osoittautunut toimivimmaksi tavaksi hankkia tietoa, huolimatta siitä kuinka vastenmielisinä sen menetelmät ja tulokset ihmiset kokisivat.

      "Evoluutio ei ole persoona. Sillä ei ole aivoja, älyä, viisautta, tietoa eikä taitoa luoda eikä kehittää yhtään mitään. Sillä ei ole viisautta luoda eikä kehittää ainuttakaan kemiallista reaktiota, joita evoluutioteoria pitää tukipilareinaan.
      Evoluutioon uskominen on siten kaiken järjen ja tieteen vastaista."

      Evoluutio on viisaudesta riippumaton kausaalinen ilmiö, mikä on sopivien olosuhteiden vallitessa luonnollista jatkumoa esim. tähtien nukleosynteesille ja erikokoisten tähtien kehityskaarille, planetaariselle akkreetiolle ja terrestristen taivaankappaleiden magmaattiselle differentiaatiolle sekä muille endogeenisille ja eksogeenisille prosesseille.

      Fysiikaaliset lainalaisuudet sanelevat energiaminimeihin pyrkivien systeemien pärinän ilmiöinä, joita inhimillinen äly voi kategorisoida vaikka millä tavalla vailla tarvetta ID:hen. Tieteessä pyritään ensin havainnoimaan ja sitten siten teoretisoimaan. Kyseessä ei ole kummoisempi puolueellisuus minkään puolesta tai vastaan.

    • 436666666666666666

      Kretiinit ovat edelleen ihmeissään etsiessään tarkoitusta sanalle fakta. Tosiasiat kun ovat niiiin hämmentäviä.

    • kielipelinne

      "Tieteessä pyritään ensin havainnoimaan ja sitten siten teoretisoimaan. Kyseessä ei ole kummoisempi puolueellisuus minkään puolesta tai vastaan. "

      Miksi tähän havainnointiin ei kelpaa reaalimaailmassa joka päivä nähtävissä oleva älykäs suunnittelu ja työ? Älykkäät ja vähemmän älykkäät ihmiset suunnittelevat ja rakentavat kaikenlaista.
      Miksi evolutionistit ovat puolueellisia ja hylkäävät älykkään suunnittelun mahdollisuuden universumin ja elämän syntymisessä?
      Leikittekö te vain sanoilla? Te näytätte olevan älyllisesti epärehellisiä ja pitäydytte tiukasti omissa päätelmissänne. Te olette AINA älykästä suunnittelua vastaan. Havainnotkin väännetään väkisin evoluutioteorian taakse kielimanipulaation keinoin.

      • Ihmisten rakentamat härvelit eivät puolla mitenkään kreationismia. Evoluutioteoria ei käsittele universumin syntyolosuhteita eikä elämän syntyä. Kommenttisi sanoilla leikkimisestä on paskanjauhantaa - mitään muuta ei kreationisteilta tähän mennesä ole tosin tullutkaan.


      • tieteenharrastaja

        Mitä tuossa jatkuvasti tankkaat selviä asioita:

        "Älykkäät ja vähemmän älykkäät ihmiset suunnittelevat ja rakentavat kaikenlaista."

        Totta kai, jokainen tuon ymmärtää.

        "Miksi evolutionistit ovat puolueellisia ja hylkäävät älykkään suunnittelun mahdollisuuden universumin ja elämän syntymisessä?""

        Mahdollisuutta eivät tietenkään hylkää. Eivät vain ole vielä havainneet objektiivista näyttöä mistään muusta kuin ihmisen tekemästä älykkäästä suunnittelusta. Tai siitä tekijästä. Tiede ei lähde juoksemaan spekulaatioiden perään ilman havaintoja. Se kuuluu uskonnon tehtävään.


      • Vapaa evokkiuskosta
        tieteenharrastaja kirjoitti:

        Mitä tuossa jatkuvasti tankkaat selviä asioita:

        "Älykkäät ja vähemmän älykkäät ihmiset suunnittelevat ja rakentavat kaikenlaista."

        Totta kai, jokainen tuon ymmärtää.

        "Miksi evolutionistit ovat puolueellisia ja hylkäävät älykkään suunnittelun mahdollisuuden universumin ja elämän syntymisessä?""

        Mahdollisuutta eivät tietenkään hylkää. Eivät vain ole vielä havainneet objektiivista näyttöä mistään muusta kuin ihmisen tekemästä älykkäästä suunnittelusta. Tai siitä tekijästä. Tiede ei lähde juoksemaan spekulaatioiden perään ilman havaintoja. Se kuuluu uskonnon tehtävään.

        Vanhimmat kaupungit ovat alle 10000 vuotta vanhoja kuten Raamattu kertoo.
        Lisäksi monien kulttuurien kertomukset lohikäärmeistä/ dinosauruksista ja vedenpaisumuksesta.


      • tieteenharrastaja
        Vapaa evokkiuskosta kirjoitti:

        Vanhimmat kaupungit ovat alle 10000 vuotta vanhoja kuten Raamattu kertoo.
        Lisäksi monien kulttuurien kertomukset lohikäärmeistä/ dinosauruksista ja vedenpaisumuksesta.

        Maa ja ihmiset ovat tietenkin paljon vanhenmpia kuin vanhimmat kaupungit.

        Kulttuurien kertomukset ovat kuulopuhetta, joka usein on dokumentoitu vasta enintään muutama sata vuotta sitten.

        Eikä noilla pakoliikkeilläsi ole tietenkään mitään yhteyttä keskusteluun, jota olit kanssani käyvinäsi.


      • >Miksi evolutionistit ovat puolueellisia ja hylkäävät älykkään suunnittelun mahdollisuuden universumin ja elämän syntymisessä?

        Ööh, miten on mahdollista, että et vielä tätä tiedä (jos siis "evolutionisteilla" tarkoitat sitä mitä luulen; olisi hyvä käyttää terminologiaa jonka kaikki ymmärtävät yksiselitteisesti)? Valuvatko toisten kommentit silmistäsi ja korvistasi huitsin nevadaan kuin vesi hanhen selästä?

        Paina nyt siis lopultakin mieleesi, että älykäs suunnittelu jätetään tieteessä huomioon ottamatta vain tasan niin kauan kuin siitä ei ole tieteellistä näyttöä eikä hetkeäkään kauempaa. ID:n kannattajat voivat antaa tuon näytön vaikka tällä minuutilla, jos sitä on, mutta kun ei vain ole.

        Minulla ei ainakaan olisi mitään sitä vastaan että näyttöä löytyisi koska minulla ei ole kerrassaan mitään älykästä suunnittelijaakaan vastaan, hänhän olisi epäilemättä tosi hyvä tyyppi, joten lopettakaa loputon, turha seliseli-lässytys ja hoitakaa hommanne!


      • kuka juoksee
        tieteenharrastaja kirjoitti:

        Mitä tuossa jatkuvasti tankkaat selviä asioita:

        "Älykkäät ja vähemmän älykkäät ihmiset suunnittelevat ja rakentavat kaikenlaista."

        Totta kai, jokainen tuon ymmärtää.

        "Miksi evolutionistit ovat puolueellisia ja hylkäävät älykkään suunnittelun mahdollisuuden universumin ja elämän syntymisessä?""

        Mahdollisuutta eivät tietenkään hylkää. Eivät vain ole vielä havainneet objektiivista näyttöä mistään muusta kuin ihmisen tekemästä älykkäästä suunnittelusta. Tai siitä tekijästä. Tiede ei lähde juoksemaan spekulaatioiden perään ilman havaintoja. Se kuuluu uskonnon tehtävään.

        " Tiede ei lähde juoksemaan spekulaatioiden perään ilman havaintoja. Se kuuluu uskonnon tehtävään. "

        Evoluutioteoria se vasta onkin spekulatiivista. 1) Sattumanvarainen itsestään syntyminen 2) Sattumanvarainen kehitys ilman päämäärää 3) Luonnonvalinta jossa askeleet ovat ainoastaan evoluutiota edistäviä.
        Nämä nyt vaan esimerkkeinä.
        Tieteenhistoria vahvistaa sen, että monet spekulatiiviset teoriat (esim. se flogiston) on jouduttu hylkäämään. Evoluutioteoriassakin on hylätty vanhentuneina pangeneesi (Darwinin itsensä esittämä ajatus) ja ortogeneesi joka ei oikein istu uusdarwinistien ajatteluun.


      • tieteenharrastaja
        kuka juoksee kirjoitti:

        " Tiede ei lähde juoksemaan spekulaatioiden perään ilman havaintoja. Se kuuluu uskonnon tehtävään. "

        Evoluutioteoria se vasta onkin spekulatiivista. 1) Sattumanvarainen itsestään syntyminen 2) Sattumanvarainen kehitys ilman päämäärää 3) Luonnonvalinta jossa askeleet ovat ainoastaan evoluutiota edistäviä.
        Nämä nyt vaan esimerkkeinä.
        Tieteenhistoria vahvistaa sen, että monet spekulatiiviset teoriat (esim. se flogiston) on jouduttu hylkäämään. Evoluutioteoriassakin on hylätty vanhentuneina pangeneesi (Darwinin itsensä esittämä ajatus) ja ortogeneesi joka ei oikein istu uusdarwinistien ajatteluun.

        Evoluutioteoria on luonnonhavaintojen selittämistä. Se ei käsittele lainkaan itsestään syntymistä., ei sen enemmän pan- kuin ortogeneesiäkään.

        Evoluution päämäärästä ei ole havaintoja, ja luonnonvalinnan askelet edistävät ainoastaan suvun jatkumista jälkeläisissä.

        Tieteessä joudutaan monesti hylkäämään spekulaatioita ja jopa hypoteeejakin. Useimmiten niitä kuitenkin vain täydennetään uusilla havainnoilla ja oivalluksilla.


      • Totuudentorvettaja
        kuka juoksee kirjoitti:

        " Tiede ei lähde juoksemaan spekulaatioiden perään ilman havaintoja. Se kuuluu uskonnon tehtävään. "

        Evoluutioteoria se vasta onkin spekulatiivista. 1) Sattumanvarainen itsestään syntyminen 2) Sattumanvarainen kehitys ilman päämäärää 3) Luonnonvalinta jossa askeleet ovat ainoastaan evoluutiota edistäviä.
        Nämä nyt vaan esimerkkeinä.
        Tieteenhistoria vahvistaa sen, että monet spekulatiiviset teoriat (esim. se flogiston) on jouduttu hylkäämään. Evoluutioteoriassakin on hylätty vanhentuneina pangeneesi (Darwinin itsensä esittämä ajatus) ja ortogeneesi joka ei oikein istu uusdarwinistien ajatteluun.

        1) Ei liity evoluutioteoriaan
        2) Et ymmärrä mistä puhut
        3) kts. vastaus kohtaan 2)

        Tiedätkö että se näyttää vähän tyhmältä jos selittää kauheasti asiasta, josta ei mitään tiedä?


      • Totuudentorvettaja kirjoitti:

        1) Ei liity evoluutioteoriaan
        2) Et ymmärrä mistä puhut
        3) kts. vastaus kohtaan 2)

        Tiedätkö että se näyttää vähän tyhmältä jos selittää kauheasti asiasta, josta ei mitään tiedä?

        >Tiedätkö että se näyttää vähän tyhmältä jos selittää kauheasti asiasta, josta ei mitään tiedä?

        No mutta Jaakobhan on meille ilmoittanut jotain siihen tapaan – eikä suinkaan ainoana kretuna – että Jumalan silmissä ihmisen viisaus on hulluutta ja hulluus viisautta. Tämä selittää suurimman osan kreationistien kommenteista.


    • Fiksu kreationisti

      - DNA
      - Ihminen
      - Eläimet
      - Kukin laji lisääntyy vain oman lajinsa sisällä
      - Iso osa eläimistä syö edelleenkin kasviksia pääasiallisena ravintonaan, kuten Raamatussa sanotaankin
      - Syntiinlankeemuksen mukanaan tuoma rappio ja seuraukset
      - Kuolema
      - Bakteerimoottori

      • Totuudentorvettaja

        En vastaa provoihin. Jos sinulla on mielessäsi jokin todiste tai fakta kreationismin puolesta, niin kerro se/ne lähteineen ja selitä, miksi asia mielestäsi todistaa kreationismin puolesta.

        Syntiinlankeemuskohtaa lukuunottamatta kaikki listasi asiat ovat evoluutioteoriaa. Tosin sen virheen teit, että kyllä eläimet lisääntyvät lajinsa ulkopuolellakin, esimerkiksi tiikerit ja leijonat keskenään.


    • sanakirja tietää

      "Tosin sen virheen teit, että kyllä eläimet lisääntyvät lajinsa ulkopuolellakin, esimerkiksi tiikerit ja leijonat keskenään. "

      Nytpä teit itse päättelyvirheen, hankipa itsellesi Biologian sanakirja ja katso sieltä "lajin" määritelmä. Voinhan minä helpottaa työtäsi ja lainaan sitä kirjaa:
      "laji, species, mon. species (lat. species=laji) eliölaji; keskenään tosiasiallisesti ja potentiaalisesti lisääntyvien yksilöiden ja populaatioiden ryhmä..".

      Ne eliöt ovat SAMAA lajia, jotka voivat tosiasiallisesti tai potentiaalisesti (mahdollisesti) lisääntyä keskenään. Esimerkkinä nyt vain tuo leijonan ja tiikerin risteymä. Myös hevosen ja aasin risteymä; muuli on hevosen lajiin kuuluva.
      Hanki ensin itsellesi oikeat tiedot ja kirjoita vasta sitten. Älä tee noita ikiomia tyhmiä päätelmiä, pystyään nyt näissä lajin virallisissa sanakirjan määrityksissä.
      Keskenään lisääntyvät omat biologian mukaan samaa lajia! Tutustu nyt edes hieman lisääntymisbiologian yksityiskohtiin ja unohda ne omat surkeat päätelmäsi.

      • Totuudentorvettaja

        "Yleisimmin käytetty lajikäsite on Ernst Mayrin määrittelemä biologinen lajikäsite, jonka mukaan eliöt kuuluvat samaan lajiin, jos ne pystyvät luonnonoloissa tuottamaan lisääntymiskykyisiä jälkeläisiä keskenään. Tämän määritelmän mukaan hevonen ja aasi ovat eri lajeja, sillä niiden risteymät eli muulit ovat lisääntymiskyvyttömiä.

        Biologisen lajikäsityksen etu on, että se pyrkii tunnistamaan sellaisia yksiköitä, jotka eivät ole vain ihmisen mielikuvituksen tuotetta, vaan joilla on todellista biologista merkitystä. "

        http://fi.wikipedia.org/wiki/Laji

        "Urosliikerit ovat steriilejä, mutta naaraspuoliset liikerit voivat lisääntyä."

        Tiikerit ja leijonat ovat siis sellaisia "välimuotoja" jotka ovat vielä niin lähellä yhteistä muotoa, että ne kykenevät lisääntymään keskenään, mutta ero kasvanut niin suureksi että ensivaiheessa on menetetty biologisesta lajikäsityksestä tutusta "kyky tuottaa lisääntymiskykyisiä jälkeläisiä" asiasta urosten lisääntymiskyky. Jos tiikerit ja leijonat pysyvät lajeina vielä pitkään elossa, seuraavaksi naaraat menettävät kyvyn myös ja vielä pidemmälle mentäessä voidaan unohtaa liikerit ja teijonat kokonaan - jos siis valintapainetta esiintyy ja lajit erkanevat entisestään toisistaan.


    • päättele oikein

      Evolutionistit hankkikaa parempi tieto lisääntymisbiologiasta. Älkää jatkuvasti sekoitelko käsitteitä genotyyppi ja fenotyyppi (ilmiasu).
      Otetaan yksi esimerkki ilmiasun vaihtelusta: mäyräkoira ja Tanskan doggi ovat molemmat koiria (siis lajina) vaikka niiden ulkoasu (ilmiasu) poikkeaa toisistaan paljonkin. Seuraappa ulkona koirien kävelyttäjiä, kyllä ne koirat tuntevat oman lajinsa vallan hyvin vaikka niiden ulkoasu on erilainen.
      Ehkä teitä pitää vielä auttaa tässäkin: Afrikassa ja Aasiassa on erinäköisiä kissaeläimiä: leijonia ja tiikereitä. Näiden syntyminen on varsin helppo selittää, alueellisesti syntyy erilaisia ilmiasuja kun pariutumiskumppaneiden luku huomioidaan. Aasiassa on ollut leijonia mutta ne on ilmeisesti metsästetty sukupuuttoon tai ne eivät ole menestyneet pariutumisessa? Ottakaa vielä huomioon se, että "albino" tiikerien ja leijonien paritumismahdollisuudet ovat heikkoja omissa elinympyröissä. Leijonauros tappaa naaraan poikaset jotta pääsee itse parittelemaan, leijonauros ei normaalisti ryhdy "ottovanhemmaksi".
      Lajikäyttäytyminen vaikuttaa paljonkin.
      Teidän tietonne on puutteellisia, te kirjoitatte vain sitä väittelytekstiä mikä löytyy evoluutiokirjoista. Miksi luonnonvalinta ei ole tuonut teille parempaa päättelykykyä?

      • >Evolutionistit hankkikaa parempi tieto lisääntymisbiologiasta.

        Koko ajan, koko ajan, neitiseni. Itse asiassa vain "evolutionistit" ylipäätään tutkivat biologian ilmiöitä tieteen kriteerein ja ovat siten kehittäneet koko nykybiologian, joten neidin ei kannata olla lainkaan huolissaan.

        >Ehkä teitä pitää vielä auttaa tässäkin: Afrikassa ja Aasiassa on erinäköisiä kissaeläimiä: leijonia ja tiikereitä. Näiden syntyminen on varsin helppo selittää, alueellisesti syntyy erilaisia ilmiasuja kun pariutumiskumppaneiden luku huomioidaan.

        Autetaanpa neitiä hieman: mitä muuta olennaista eroa leijonalla ja tiikerillä on kuin ulkonäkö?


    • valistunut neiti

      "Autetaanpa neitiä hieman: mitä muuta olennaista eroa leijonalla ja tiikerillä on kuin ulkonäkö?"

      Kiitos avun tarjouksesta, mutta tältä osin se ei nyt ole aivan ajankohtaista koska "neiti" opiskelee biologiaa ja tenteissäkin on pärjätty hyvin. Kyllä minä tiedän leijonan ja tiikerin eron. Ja tunnistan myös omassa kotikissassani kissaeläimille tyypillisen käyttäytymisen.
      Kissoilla on oma biologia siinä missä niillä on oma käyttätymisensä. Kissat käyttäytyy eri tavalla kuin koiraeläimet.
      Kyllä tässä osataan erotella eri tieteiden alueet; biologia, etologia ja ekologia. Ja osataan vielä ajatella "poikkitieteellisesti". Ehkä tässä pitäisikin auttaa teitä?

      • Eli et kuitenkaan tiedä, kun tekstisi kertoo että et tiedä?

        Minäpä kerron jotain "biologian opiskelijalle":

        Leijona on laumaeläin, tiikeri yksineläjä. Tämä on aivan olennainen ero.
        Tiikerin lihasmassa on suurempi, suurimmillaan jopa paljon.
        Leijona elää aukeilla alueilla, mutta tiikeri lähinnä metsissä ja tiheiköissä.

        Jos kerran olet fiksu, sen voi näyttää myös kirjoittelussaan ihan reilusti. Nimimerkin pysyvyydestä voisi aloittaa.

        Arvaapa moniko uskoo sinun oikeasti opiskelevan biologiaa? Ja molemmat ovat kreationisteja.


      • lue uudelleen
        RepeRuutikallo kirjoitti:

        Eli et kuitenkaan tiedä, kun tekstisi kertoo että et tiedä?

        Minäpä kerron jotain "biologian opiskelijalle":

        Leijona on laumaeläin, tiikeri yksineläjä. Tämä on aivan olennainen ero.
        Tiikerin lihasmassa on suurempi, suurimmillaan jopa paljon.
        Leijona elää aukeilla alueilla, mutta tiikeri lähinnä metsissä ja tiheiköissä.

        Jos kerran olet fiksu, sen voi näyttää myös kirjoittelussaan ihan reilusti. Nimimerkin pysyvyydestä voisi aloittaa.

        Arvaapa moniko uskoo sinun oikeasti opiskelevan biologiaa? Ja molemmat ovat kreationisteja.

        "Kyllä minä tiedän leijonan ja tiikerin eron. Ja tunnistan myös omassa kotikissassani kissaeläimille tyypillisen käyttäytymisen."

        Unohditko lukea tuon? Leijona on laumaeläin ja tiikeri on yksineläjä, nuohan ovat juuri niitä käyttäytymiseroja! Miten voikaan olla noin huono lukutaito?
        Se leijonan ja tiikerin risteymä syntyi eläintarhassa eli poissa noiden isojen kissojen luontaisesta elämän ympäristöstä. Tajusitko tätäkään kissojen käyttäytymisessä? Tarkistahan oma päättelykykysi ennenkuin ryhdyt arvostelemaan toisia.


      • lue uudelleen kirjoitti:

        "Kyllä minä tiedän leijonan ja tiikerin eron. Ja tunnistan myös omassa kotikissassani kissaeläimille tyypillisen käyttäytymisen."

        Unohditko lukea tuon? Leijona on laumaeläin ja tiikeri on yksineläjä, nuohan ovat juuri niitä käyttäytymiseroja! Miten voikaan olla noin huono lukutaito?
        Se leijonan ja tiikerin risteymä syntyi eläintarhassa eli poissa noiden isojen kissojen luontaisesta elämän ympäristöstä. Tajusitko tätäkään kissojen käyttäytymisessä? Tarkistahan oma päättelykykysi ennenkuin ryhdyt arvostelemaan toisia.

        " Leijona on laumaeläin ja tiikeri on yksineläjä, nuohan ovat juuri niitä käyttäytymiseroja! "

        Hmm, kuitenkin tuosta viestistäsi kerrot, että kotikissallasi on kissaeläimille lajityypillinen käyttäytyminen - mutta et kerro mikä se kissaeläimille lajityypillinen käyttäytyminen on : ""Kyllä minä tiedän leijonan ja tiikerin eron. Ja tunnistan myös omassa kotikissassani kissaeläimille tyypillisen käyttäytymisen.""

        Onko siis kissaeläimille lajityypillistä olla laumaeläin, kuten leijonalla on, vaiko yksineläjä, kuten tiikerillä on?

        Joten ainakin minusta täältä sivusta seuraten näyttää siltä, että sinun oma logiikkasi ei oikein toimi ja viestintäsi antaisi syytä epäillä sinua trolliksi. Varsinkin kun vaihdat nikkiä trollityypilliseen tapaan.


    • "Kiitos avun tarjouksesta, mutta tältä osin se ei nyt ole aivan ajankohtaista koska "neiti" opiskelee biologiaa ja tenteissäkin on pärjätty hyvin. Kyllä minä tiedän leijonan ja tiikerin eron."

      Osasitko myös vastata tentissä, että ne ovat eri lajeja?

      • Vapaa evokkiuskosta

        Raamatussa jumala ei luonut radioaktiivisia aineita. Radioaktiivisuus voi kasvaa todella nopeasti, joten on oletettavaa, että alussa raidioaktiivisuus kasvoi mopeasti. Evokit ovatkin tehneet mittauksia, joissa alimmissa maakerroksissa on vähän radioaktiivisuutta.
        Raamatussa maanviljely alkoi heti paratiisista karkoituksen jälkeen. Evokkien mukaan viljely alkoi 10000ekr. Noin 4000 vuoden eron selittää radioaktiivisuuden nousu, jota evokit eivät huomio.


      • Vapaa evokkiuskosta kirjoitti:

        Raamatussa jumala ei luonut radioaktiivisia aineita. Radioaktiivisuus voi kasvaa todella nopeasti, joten on oletettavaa, että alussa raidioaktiivisuus kasvoi mopeasti. Evokit ovatkin tehneet mittauksia, joissa alimmissa maakerroksissa on vähän radioaktiivisuutta.
        Raamatussa maanviljely alkoi heti paratiisista karkoituksen jälkeen. Evokkien mukaan viljely alkoi 10000ekr. Noin 4000 vuoden eron selittää radioaktiivisuuden nousu, jota evokit eivät huomio.

        >Raamatussa jumala ei luonut radioaktiivisia aineita.

        Tämän on oltava vitsi. Onhan se?


      • tentit läpi

        "Osasitko myös vastata tentissä, että ne ovat eri lajeja? "

        Tuota ei kylläkään ole tentissä kysytty. Miten itse olet tenteissä vastannut?


      • RepeRuutikallo kirjoitti:

        >Raamatussa jumala ei luonut radioaktiivisia aineita.

        Tämän on oltava vitsi. Onhan se?

        Pakko olla. Jos Raamatun Jumala ei luonut radioaktiivisuutta, niin kukas sen loi?

        Ei kait se syntynyt itsekseen?


      • tentit läpi kirjoitti:

        "Osasitko myös vastata tentissä, että ne ovat eri lajeja? "

        Tuota ei kylläkään ole tentissä kysytty. Miten itse olet tenteissä vastannut?

        Vaikka sitä ei tentissä kysytty, niin osaisitko vastata myös kysyttäessä, että ne ovat eri lajeja?

        Ovatko ne eri lajeja?


      • tentit läpi kirjoitti:

        "Osasitko myös vastata tentissä, että ne ovat eri lajeja? "

        Tuota ei kylläkään ole tentissä kysytty. Miten itse olet tenteissä vastannut?

        "Tuota ei kylläkään ole tentissä kysytty."

        Eipä tietenkään, koska opettaja ei edes voisi kuvitella, että joku luulee niitä samoiksi lajeiksi sen vuoksi, että on hirttäytynyt biologian sanakirjan määritelmän sanaan potentiaalisesti. Maailmasta ei löydy yhtään biologia, joka luulisi niitä samoiksi lajeiksi huolimatta siitä, että ne voivat saada hybridin tarhaolosuhteissa.

        "Miten itse olet tenteissä vastannut?"

        Ihan riittävän hyvin, kiitos kysymästä.


      • Vapaa evokkiuskosta kirjoitti:

        Raamatussa jumala ei luonut radioaktiivisia aineita. Radioaktiivisuus voi kasvaa todella nopeasti, joten on oletettavaa, että alussa raidioaktiivisuus kasvoi mopeasti. Evokit ovatkin tehneet mittauksia, joissa alimmissa maakerroksissa on vähän radioaktiivisuutta.
        Raamatussa maanviljely alkoi heti paratiisista karkoituksen jälkeen. Evokkien mukaan viljely alkoi 10000ekr. Noin 4000 vuoden eron selittää radioaktiivisuuden nousu, jota evokit eivät huomio.

        "Raamatussa jumala ei luonut radioaktiivisia aineita."

        Niin. Eivät nuo Raamatun kirjoittaneet pimentolaiset voineet edes kuvitella mitään tuollaista luonnonilmiötä.

        "Radioaktiivisuus voi kasvaa todella nopeasti, joten on oletettavaa, että alussa raidioaktiivisuus kasvoi mopeasti."

        Se ei voi kasvaa, koska se perustuu muuttumattomiin luonnonvakioihin. Ja mittaukset ovatkin osoittaneet, ettei se ole kasvanut, vaan pysynyt vakiona miljardeja vuosia. Vain jotkin tietyt erityisolosuhteet voivat saada muutamia isotooppeja hajoamaan nopeammin.

        "Evokit ovatkin tehneet mittauksia, joissa alimmissa maakerroksissa on vähän radioaktiivisuutta."

        Tarkoitatko, että vanhimmissa kerroksissa on vähemmän radioaktiivisuutta? Se johtuu siitä, että miljardien vuosien aikana suuri osa radioaktiivisesta aineksesta on jo ehtinyt hajota.

        "Raamatussa maanviljely alkoi heti paratiisista karkoituksen jälkeen."

        Maanviljelystä ihminen alkoi harjoittaa satoja tuhansia vuosia kehityksensä jälkeen.

        "Evokkien mukaan viljely alkoi 10000ekr. Noin 4000 vuoden eron selittää radioaktiivisuuden nousu, jota evokit eivät huomio."

        Miten voisimme sellaisen huomioida, kun se on luonnonlakien vastainen mahdottomuus ja mittaukset osoittavat ettei sellaista ole tapahtunut?


      • KATin haamu
        Vapaa evokkiuskosta kirjoitti:

        Raamatussa jumala ei luonut radioaktiivisia aineita. Radioaktiivisuus voi kasvaa todella nopeasti, joten on oletettavaa, että alussa raidioaktiivisuus kasvoi mopeasti. Evokit ovatkin tehneet mittauksia, joissa alimmissa maakerroksissa on vähän radioaktiivisuutta.
        Raamatussa maanviljely alkoi heti paratiisista karkoituksen jälkeen. Evokkien mukaan viljely alkoi 10000ekr. Noin 4000 vuoden eron selittää radioaktiivisuuden nousu, jota evokit eivät huomio.

        Radioaktiivisuus näkyy miljoonien jopa miljardien vuosien takaa suorana lähetyksenä (tähtien ja galaksien valossa).

        Keväällä katsottiin tähtikerhossa kaukoputkella Otavan galaksin M82:n räjähtäneen tähden eli supernovan radioaktiivisen nikkelin(Ni-56) ja koboltin(Co-56) hehkua joka lähti 11,5 miljoonaa vuotta sitten liikenteeseen.

        http://en.wikipedia.org/wiki/SN_2014J


    • reseptin sisältö

      Voiko suklaamuffinsin reseptillä tuottaa sacherkakkua? Ei, koska resepti on siihen liian yksinkertainen.
      Miten siis alun kemiallinen evoluutio tuotti ensimmäisen elävän esisolun? Yksinkertaiset reseptit eivät muutu itsestään monimutkaisemmiksi. Kyllä se on se leipuri joka valitsee oikean reseptin halutessaan tuottaa niitä kakkuja joita tarkoittaa.

      • Luojalla oli siis reseptikirja. Kukahan sen oli tehnyt?


      • KATin haamu

        Parempia reseptejä löytyy myös sattumalta kun sattuma pudottaa aineksensa joukkoon tai häiritsee niin että tulee väärää ainesta joukkoon.

        Vai eikö kokkeja koskaan ole käsketty KOKEILEMAAN uutta ja tuntematonta ?


      • KATin haamu
        KATin haamu kirjoitti:

        Parempia reseptejä löytyy myös sattumalta kun sattuma pudottaa aineksensa joukkoon tai häiritsee niin että tulee väärää ainesta joukkoon.

        Vai eikö kokkeja koskaan ole käsketty KOKEILEMAAN uutta ja tuntematonta ?

        1800-luvulla Goodyearin kerrotaan kehittäneen kumin vulkanisoinnin sattuman kautta. Rengasresepti sattuman tulos :D


      • Eieiei,
        KATin haamu kirjoitti:

        1800-luvulla Goodyearin kerrotaan kehittäneen kumin vulkanisoinnin sattuman kautta. Rengasresepti sattuman tulos :D

        Kretut ei usko sattumaan! Ne vaan sulkee korvansa ja huutaa: "Eieieieieieieieiei..."


    Ketjusta on poistettu 0 sääntöjenvastaista viestiä.

    Luetuimmat keskustelut

    1. Heikki Silvennoinen petti vaimoaan vuosien ajan

      Viiden lapsen isä Heikki kehuu kirjassaan kuinka paljon on pettänyt vaimoaan vuosien varrella.
      Kotimaiset julkkisjuorut
      146
      2550
    2. Taasko se show alkaa

      Koo osottaa taas mieltään
      Ikävä
      24
      1986
    3. Miksi ihmeessä nainen seurustelit kanssani joskus

      Olin ruma silloin ja nykyisin vielä rumempi En voi kuin miettiä että miksi Olitko vain rikki edellisestä suhteesta ja ha
      Ikävä
      23
      1968
    4. Persut nimittivät kummeli-hahmon valtiosihteeriksi!

      Persujen riveistä löytyi taas uusi törkyturpa valtiosihteeriksi! Jutun perusteella järjenjuoksu on kuin sketsihahmolla.
      Perussuomalaiset
      89
      1776
    5. Onko ministeri Juuso epäkelpo ministerin tehtäviensä hoitamiseen?

      Eikö hänellä ole kompetenttia hoitaa sosiaali- ja terveysministetin toimialalle kuuluvia ministerin tehtäviä?
      Perussuomalaiset
      67
      1530
    6. Sakarjan kirjan 6. luku

      Jolla korva on, se kuulkoon. Sain profetian 22.4.2023. Sen sisältö oli seuraava: Suomeen tulee nälänhätä niin, että se
      Profetiat
      20
      1306
    7. Avaa sydämesi mulle

      ❤ ❤❤ Tahdon pelkkää hyvää sulle Sillä ilmeisesti puhumalla Avoimesti välillämme Kaikki taas selviää Kerro kaikki, tahdo
      Ikävä
      37
      1202
    8. Elia tulee vielä

      Johannes Kastaja oli Elia, mutta Jeesus sanoi, että Elia tulee vielä. Malakian kirjan profetia Eliasta toteutuu kokonaan
      Helluntailaisuus
      36
      1188
    9. Söpö lutunen oot

      Kaipaan aina vaan, vaikkakin sitten yksipuolisesti.
      Ikävä
      11
      1188
    10. Nellietä Emmaa ja Amandaa stressaa

      Ukkii minnuu Emmaa ja Amandaa stressaa ihan sikana joten voidaanko me koko kolmikko hypätä ukin kainaloon ja syleilyyn k
      Isovanhempien jutut
      10
      1167
    Aihe